FM Assessment
A 4-week-old male infant presents with persistent conjunctivitis that started two weeks ago. His mother reports excessive tearing "that won't stop." He is up to date on immunizations. On physical examination, the patient is well-appearing, well-nourished, and well-hydrated. He is noted to have excessive bilateral tearing. Dacryostenosis is suspected. Which of the following is the first-line treatment for this patient? ABalloon dacryoplasty BCrigler massage CLacrimal duct intubation DLacrimal duct probing
BCrigler massage Explanation: Nasolacrimal duct obstruction (also called dacryostenosis) is obstruction of the nasolacrimal duct resulting in excessive tearing. It may be congenital or acquired. Congenital dacryostenosis typically manifests as tearing, purulent discharge, mattering (debris on the eyelashes) or chronic conjunctivitis (rare) with onset after the first two weeks of life. Causes of acquired nasolacrimal duct obstruction include sinus surgery, prior nasal or facial bone fractures, sarcoidosis, and age-related stenosis of the nasolacrimal duct. Diagnosis is usually clinical. The first-line treatment of congenital dacryostenosis is Crigler massage (lacrimal sac massage) and observation. Cases that persist to 6 to 10 months of age despite Crigler massage should be treated with simple lacrimal duct probing surgical procedure. For older children, nasolacrimal duct obstruction intubation is performed under general anesthesia. More invasive surgical management options including balloon dacryoplasty, dacryocystorhinostomy, and conjunctivodacryocystorhinostomy may be performed in patients who have failed treatment with less invasive interventions. In acquired dacryostenosis, the underlying illness should be treated. If treatment is not possible or ineffective, invasive surgical intervention should be considered. Antibiotics may be helpful in cases due to bacterial overgrowth. Spontaneous resolution occurs in more than 90% of patients with congenital nasolacrimal duct obstruction.
A 55-year-old man has been recently treated with radiation therapy for Stage 1 squamous cell carcinoma of the mandible. Which of the following would be the best recommendation for the maintenance of osteoradionecrosis? ABotulinum toxin BDental referral CPentoxifylline DSpeech therapy
BDental referral Explanation: Head and neck cancers are most commonly caused by squamous cell carcinoma. Surgery or radiation therapy are the treatment modalities for early stage findings while chemotherapy is added in later stages. Xerostomia is the most common complication in patients who have undergone radiation therapy and is often a long-term complication of treatment. This condition can lead to the deterioration of the dentition and oral cavity. A decrease in saliva can increase the acidity of the mouth, which demineralizes the dentition. An increase in decay may require extractions or the need for dental restorations. Osteoradionecrosis is seen in patients who need extractions or dental restoration in areas that have been previously treated with radiation therapy. It is caused by a decrease in vascularity of the bone and surrounding tissues secondary to radiation therapy. When a tooth requires extraction in an area that is hypovascular, the area may not heal. Osteomyelitis, trismus, fistulas, and pain can all occur due to this condition. Treatment includes antibiotics, debridement or resection, and hyperbaric oxygen. A dental referral to prevent decay and maintain the oral cavity is essential in the maintenance of patients who have undergone radiation therapy. In addition to xerostomia, there are other complications, such as mucositis, where a dental referral would be a beneficial recommendation for improving a patient's quality of life.
A 25-year old woman at 10 weeks gestation presents to the office with complaints of nausea and vomiting. She states she has tried altering her diet, eating smaller meals without relief. She states she is nauseous throughout the day and typically vomits at least twice. Her vital signs are within normal limits and she is not experiencing any orthostasis. Her physical exam is benign. Which of the following medications would be most appropriate to prescribe at this time? ADiphenhydramine BDoxylamine-pyridoxine COndansetron DPromethazine
BDoxylamine-pyridoxine Explanation: Nausea and vomiting are common in pregnant women, particularly early in pregnancy; however, severe vomiting can result in complications such as hypovolemia and weight loss. If a woman has a past medical history significant for migraines, motion sickness, or nausea with estrogen-based medications, then she is more at risk for developing nausea with or without vomiting during pregnancy. Possible etiologies for nausea and vomiting during pregnancy include elevated human chorionic gonadotropin (hCG) levels, elevated estrogen levels, and delayed gastric motility although no studies have definitively identified a causative agent. A genetic predisposition has been demonstrated to nausea and vomiting during pregnancy with first-degree relatives experiencing similar symptoms during pregnancy. The mean onset for nausea with pregnancy is between 5 and 6 weeks gestation with the peak of symptoms occurring around 9 weeks gestation. Typically by 20 weeks gestation, symptoms of nausea or vomiting have resolved. Nausea and vomiting can occur at any time during the day despite the commonly used term "morning sickness." Patients who have weight loss greater than 5% of their prepregnancy weight or have vomiting more than three times per day are characterized as having hyperemesis gravidarum. Women with persistent vomiting may develop orthostatic hypotension or electrolyte disturbances. Treatment includes dietary changes, avoiding triggers for nausea, and drinking fluids before meals. Pharmacologic treatment includes pyridoxine with or without doxylamine succinate. Other antihistamines, such as diphenhydramine, have been found to be effective in treating nausea as well. For women who have intractable vomiting, metoclopramide or promethazine is a short-term option.
A 56-year-old woman presents with fever, jaundice, and right upper abdominal pain. Physical examination reveals scleral icterus, generalized jaundice, and a negative Murphy sign. Laboratory studies show white blood cell count of 14,000 cells/mm3 of blood, lipase of 180 U/L, direct bilirubin of 3 mg/dL, and AST and ALT of 120 U/L and 200 U/L, respectively. A bedside abdominal ultrasound demonstrates a common bile duct measuring 1.5 cm. Which of the following is the most likely diagnosis? AAcute cholangitis BAcute cholecystitis CAcute pancreatitis DBiliary leak
A Acute cholangitis Explanation: Cholangitis is a clinical syndrome characterized by the Charcot triad of fever, jaundice, and abdominal pain that develops as a result of biliary duct obstruction and subsequent bacterial colonization. Patients classically present with fever, jaundice, and abdominal pain. In addition to the Charcot triad, patients with severe disease may present with hypotension and altered mental status, a pentad known as the Reynold pentad. Patients may also present with signs and symptoms of bacteremia such as sepsis, hepatic abscess, multiple organ system dysfunction, and shock. Laboratory studies may show leukocytosis, elevated direct bilirubin, abnormal liver enzymes, elevated alkaline phosphatase, and elevated gamma-glutamyl transpeptidase. Abdominal ultrasound typically shows a bile duct stone and bile duct dilation. Abdominal computed tomography should be obtained if the ultrasound result is normal. In patients with the Charcot triad, endoscopic retrograde cholangiopancreatography may be performed initially for confirmation and to provide biliary drainage. Treatment is supportive, along with antibiotic therapy, and biliary drainage. Broad-spectrum antibiotics (piperacillin-tazobactam) with coverage against enteric organisms must be initiated. Patients with mild to moderate cholangitis who fail to respond to conservative treatment for 24 hours, and those with severe cholangitis require urgent biliary decompression (endoscopic sphincterotomy with stone extraction and stent placement). Endoscopic retrograde cholangiopancreatography is both diagnostic and therapeutic. Prognosis has improved with advances in care, however, mortality for severe cholangitis remains high.
What is the most important risk factor for spontaneous abortion in healthy women? AAdvanced maternal age BIncreased gravidity CLow folate levels DTobacco abuse
A Advanced maternal age Explanation: Spontaneous abortion is defined as loss of a pregnancy before 20 weeks gestation. The incidence of spontaneous abortion is approximately 15%t in all pregnancies. This percentage may be higher when pregnancy loss before maternal detection is factored in. Advanced maternal age is the greatest risk factor for spontaneous abortion, however, other risk factors have been identified as well. These include tobacco abuse, alcohol abuse, and cocaine abuse. The risk of spontaneous abortion is less in women who have had previous live births. Most spontaneous abortions occur before the gestational age of 8 weeks and are secondary to chromosomal abnormalities. Symptoms consist of vaginal bleeding and pelvic cramping. Physical evaluation should include a pelvic exam to confirm uterine bleeding and examine for adnexal abnormalities. A pelvic ultrasound is useful in determining placement of the fetus, gestational and yolk sac size, fetal cardiac activity, and fetal size. The lack of normal development or fetal cardiac activity can be useful in diagnosing spontaneous abortion. Serum human chorionic gonadotropin (hCG) levels should be drawn and followed. In spontaneous abortion, the level will decrease over time. Women who are Rh(D)-negative require anti-D immune globulin if there is vaginal bleeding during early pregnancy or if spontaneous abortion occurs.
A 45-year-old man presents to the clinic with fatigue and arthralgias that have been worsening over the past few months. A physical exam reveals skin hyperpigmentation. Iron studies show elevated levels of serum iron and ferritin, as well as elevated liver function studies. Genetic testing confirms an HFE mutation. Considering the most likely diagnosis, which of the following should the patient avoid? AAlcohol BCooked shrimp CSpinach DTofu
A Alcohol Explanation: Hereditary hemochromatosis is a genetic disorder that increases intestinal iron absorption leading to iron overload in some individuals. It is an autosomal recessive disorder and not all individuals who are homozygous for the mutation will be affected. Patients who present with iron overload typically do not have symptoms until later in life when iron has been accumulating for decades. The average age at the time of diagnosis in males is 40 years of age and potentially later in females because of iron loss associated with menstruation and pregnancy. Symptoms can vary widely and may include fatigue and arthralgias. Sinus node dysfunction and dysrhythmias can occur with cardiac iron overload. The pancreas or pituitary gland could be affected as well, resulting in endocrine dysfunction. Skin hyperpigmentation may occur from iron deposition. Iron studies, liver function tests, and HFE mutation testing aids in the diagnosis. Magnetic resonance imaging (MRI) has replaced liver biopsy as the modality of choice in determining iron stores. Patients should be counseled to avoid alcohol, ascorbic acid supplements, and the consumption of raw seafood. Treatment is indicated in patients who demonstrate iron overload and consists of therapeutic phlebotomy. Complications include hepatocellular carcinoma, cirrhosis, diabetes mellitus, and cardiomyopathy.
A 24-year-old woman with endometriosis presents to the office with a complaint of frequent headaches for the past year. She reports a severe pulsating headache located above and behind her right eye with light sensitivity and nausea. Headache occurs 12 days per month. She currently takes naproxen to abort the headache with good relief. She would like to start a preventative medication. She is currently on norethindrone acetate and ethinyl estradiol. Which of the following medications would be most appropriate to initiate? AAmitriptyline 25 mg orally nightlyCorrect Answer BErenumab 70 mg subcutaneously once monthly CTopiramate 25 mg orally nightly DValproate 500 mg orally once daily
A Amitriptyline 25 mg orally nightly Explanation: Migraine headache is a headache disorder associated with nausea and light and sound sensitivity. Migraine is due to primary neuronal dysfunction causing cortical spreading depression, leading to the activation of the trigeminal nociceptive pathway, which is the primary basis for the pain associated with migraine headache. Migraine is more commonly seen in women than men and in patients who have a positive family history of migraine headache. Migraines can be triggered by stress, menstruation, barometric pressure changes, odors, wine, and particular foods, among many other possible triggers. Migraine typically has distinct phases, including prodrome, aura, headache, and postdrome. Not every phase is observed in each patient. Prodromal symptoms can include yawning, irritability, food cravings, and neck stiffness. Aura is observed in a quarter of migraineurs. Aura can be visual (scotoma, floaters, flashes of light, loss of vision), speech (slurring, aphasia), auditory (tinnitus, loss of hearing), sensory (paresthesia, numbness, burning pain), or motor (jerking, paralysis, facial droop). Visual aura is the most common. The duration of an aura is five minutes to an hour. The headache with a migraine is more often unilateral and typically pulsatile in nature. It can be moderate-to-severe and generally lasts for several hours. Nausea with or without vomiting can be observed. Photophobia or phonophobia are common. The postdrome phase includes a period of exhaustion. There are variants of migraine that are associated with a particular subtype of aura symptoms. These variants include migraine with brainstem aura, hemiplegic migraine, vestibular migraine, menstrual migraine, and retinal migraine. These variants may respond better to specific preventative therapies. Additionally, certain abortive medications, such as triptans, should be avoided in hemiplegic and brainstem aura subtypes due to the risk of cerebral vasoconstriction. The diagnosis of migraine is based on clinical history and physical examination, and a distinction should be made as to whether the patient has aura or not. Neuroimaging should be considered if the patient has abnormal physical examination findings or other atypical headache features. Preventative therapy is typically considered in patients who have more than four headaches per month lasting longer than 12 hours. Initial preventative treatment includes a tricyclic antidepressant (amitriptyline or nortriptyline), a beta-blocker (such as propranolol), or topiramate. The appropriate treatment regimen should be selected based on the patient's age, comorbidities, and use of other medications. If initial drug choice is not effective, it is recommended that the patient try an alternative drug class. Abortive migraine management is dependent on the severity of symptoms and presence of nausea or vomiting. Simple analgesics (NSAIDs, acetaminophen) are effective for mild-to-moderate migraine headaches. For moderate-to-severe migraines without nausea, an oral triptan is recommended. For moderate-to-severe migraine with nausea, subcutaneous sumatriptan, nasal sumatriptan or zolmitriptan, or dihydroergotamine is recommended. Antiemetics should be used when nausea and vomiting are present.
A 25-year-old pregnant woman presents to the clinic complaining of tremor and palpitations. She is in the ninth week of pregnancy and has no past medical history. Vitals demonstrate HR 118, RR 19, T 98.4° F, BP 122/79, and SpO2 99% on room air. Laboratory results indicate TSH 0.09 mIU/L, free T3 2.19 ng/dL, T4 67 pmol/L, elevated antithyroperoxidase antibodies, and presence of thyroid-stimulating immunoglobulin. Which of the following treatments is contraindicated? AAtenolol BPotassium iodide solution CPropylthiouracil DVerapamil
A Atenolol Explanation: Hyperthyroidism is an endocrine disorder caused by elevated serum levels of thyroid hormones (T4 and T3). While transient elevations of thyroid hormones are normal in early pregnancy due to increased metabolic demand, overt hyperthyroidism can also manifest. Nonspecific clinical manifestations that present in normal pregnancy and hyperthyroidism include tachycardia, heat intolerance, and increased perspiration. Additional hyperthyroid symptoms include tremor, anxiety, and weight loss despite normal or increased appetite. Graves disease, the most common cause of hyperthyroidism, can also result in characteristic findings such as goiter, ophthalmopathy, and pretibial myxedema. This autoimmune disorder of the thyroid gland causes increased synthesis and release of thyroid hormones secondary to stimulation by thyroid-stimulating immunoglobulins (thyroid synthesis hormone receptor antibodies). Laboratory results for hyperthyroidism indicate elevated thyroid hormone levels with decreased serum thyroid-stimulating hormone, with specific findings of Graves disease including detectable serum thyroid-stimulating immunoglobulins. Graves disease may also demonstrate elevated antithyroperoxidase or antithyroglobulin antibodies and antinuclear antibodies, but these findings are nonspecific. While transient elevations of thyroid hormones and suppressed thyroid-stimulating hormone levels are normal in early pregnancy, overt hyperthyroidism should be suspected with serum thyroid-stimulating hormone values < 0.1 mIU/L and a free T4 or free T3 greater than the normal range for pregnancy. In nonpregnant patients, radioisotope uptake and scanning can aid in diagnosing patients with thyroid disorders. This testing is contraindicated in pregnant or nursing women and alternative strategies, such as thyroid ultrasound, should be used. First-line treatment for symptoms of moderate to severe hyperthyroidism involves a beta-blocker, such as propranolol or metoprolol. Atenolol is contraindicated in pregnant patients due to intrauterine growth restriction. Antithyroid drugs include propylthiouracil and methimazole, with the former being the first-line treatment for decrease of thyroid hormone synthesis in first-trimester pregnant women. Methimazole is the preferred agent in second- and third-trimester pregnant women. Thyroidectomy can be performed in pregnant women after the second trimester but should be postponed until the postpartum time unless the disease is severe or presents with concurrent aggressive thyroid cancer. Pregnancy complications that can result from untreated overt hyperthyroidism include spontaneous abortion, premature labor, low birth weight, stillbirth, preeclampsia, and heart failure.
A 78-year-old man presents to the clinic with concerns for dyspnea on exertion and fatigue for two days. He has never experienced this in the past and denies any chest pain. His vital signs show a heart rate of 134. An electrocardiogram is completed and shows an irregularly irregular rhythm without discernible P waves. Which of the following is the most likely diagnosis? AAtrial fibrillation BMultifocal atrial tachycardia CSupraventricular tachycardia DVentricular fibrillation
A Atrial fibrillation Explanation: Atrial fibrillation is a dysrhythmia characterized by an irregularly irregular pattern of RR intervals and an absence of discernable P waves on electrocardiogram (ECG). The dysrhythmia can be persistent or paroxysmal. Paroxysmal atrial fibrillation is defined as an episode of dysrhythmia that lasts less than seven days. Hypertensive heart disease and coronary heart disease are typically the underlying pathology responsible in the development of atrial fibrillation. Many patients do not complain of symptoms; however, symptoms may include fatigue, dyspnea on exertion, tachycardia, or palpitations. Lab testing should include an evaluation of thyroid function. An echocardiogram should be done to assess the size of the atria and the function of the ventricles. A transesophageal echocardiogram (TEE) should be considered to rule out a left atrial thrombus. Treatment options include cardioversion, anticoagulation therapy, or rate controlling drugs. Percutaneous catheter ablation is an option for long-term definitive treatment. Treatment is dependent on whether the dysrhythmia is persistent or paroxysmal and on whether comorbidities exist.
A 48-year-old woman presents to a rural urgent care clinic with acute onset of shortness of breath and pleuritic chest pain. She states her right leg began to swell and became painful yesterday, and the dyspnea started twenty minutes ago. She is an obese smoker, and her only daily medication is hormone replacement therapy. Abnormal vital signs include heart rate of 120 beats/minute, blood pressure of 90/60 mm Hg, and pulse oximetry of 92% on room air. Which of the following is the most appropriate preliminary diagnostic test for this patient? ABedside Doppler ultrasound of the right lower extremity BChest X-ray CComputed tomographic pulmonary angiography DVentilation-perfusion scan
A Bedside Doppler ultrasound of the right lower extremity Explanation: Acute pulmonary embolism is a medical emergency describing embolic material in one or more bronchial blood vessels. The embolism impedes perfusion of the lung and can lead to decreased oxygenation, shock, or death. Patients at risk for pulmonary embolism are those at risk for thrombus formation. The Virchow triad states the likelihood of thrombus formation is related to stasis, endothelial damage, and hypercoagulability. Patients who are at risk of thrombus formation, deep venous thrombosis, and pulmonary embolism include those who are bedridden or immobilized, those with atherosclerosis, smokers, those taking hormone therapy, those with active cancer, victims of physical trauma, the obese, and those with genetic disorders of hypercoagulability, such as factor V Leiden and the prothrombin gene mutation. Symptoms of pulmonary embolism may range from imperceptible to sudden death and include dyspnea, pleuritic chest pain, unilateral swollen extremity, hypotension, tachycardia, hemoptysis, dysrhythmia, and syncope. Pulmonary embolism can be definitively diagnosed with computed tomographic pulmonary angiography, however, this test may not be available for all patients. Prompt treatment with antithrombotic agents is necessary to restore blood flow to the lungs. If a patient presents with symptoms of acute pulmonary embolism, a history that is consistent with the diagnosis, and a swollen lower extremity, then bedside Doppler ultrasound of the swollen extremity is an appropriate first-line diagnostic test to obtain if CT is immediately unavailable, as the presence of a deep vein thrombosis in this setting will be sufficient to initiate time-sensitive treatment with antithrombotic agents. If the clinical picture is murkier and the patient is stable, a D-dimer test should be drawn. The D-dimer test will be positive (elevated) in patients with pulmonary embolism. Treatment of pulmonary embolism depends upon the hemodynamic stability of the patient and the size of the thrombus. If a patient is hemodynamically unstable, thrombolytic therapy or embolectomy may be necessary. These patients also require oxygen therapy and may require resuscitative efforts. Hemodynamically stable patients with pulmonary embolism should be hospitalized while anticoagulation with heparin or enoxaparin is initiated. Where not contraindicated, warfarin is also begun and the patient discharged once the international normalized ratio is at therapeutic range (2.0 to 3.0). Patients who survive a first-time pulmonary embolism and have modifiable hypercoagulable risk factors should continue anticoagulant treatment for three months. Longer or indefinite anticoagulation may be necessary for patients with persistent risk factors for thrombus formation.
A 21-year-old woman presents to the clinic complaining of severely depressed mood for the past five days. Her mother adds that earlier this month she became hyperactive, with rapid speech, expansive mood, a flight of ideas, and insomnia that lasted four weeks, necessitating a leave of absence from college. She takes no medications, has a negative urine drug screen, and denies illicit drug use. Which of the following is the most likely diagnosis? ABipolar I disorder BBipolar II disorder CCyclothymia DMajor depressive disorder
A Bipolar I disorder Bipolar I disorder is a mood disorder characterized by an episode of mania lasting at least seven days or necessitating hospitalization. Symptoms of mania include expansive mood, increased irritability, decreased need for sleep, flight of ideas, rapid or pressured speech, increase in goal-directed activity, excessive engagement in pleasurable activities with potentially negative consequences, distractibility, and psychomotor agitation. Three or more of these symptoms must be present every day for most of the day, must significantly interfere with normal functioning, and must represent a distinct change from normal behavior. The symptoms cannot be attributable to psychoactive substances. At times, patients with mania will present with overt psychosis. Most patients with bipolar I disorder also suffer at least one episode of major depression, which is defined as depressed mood, suicidal ideation, anhedonia, sleeping or eating disturbances, and feelings of guilt persisting most of the day, every day for at least two weeks. A history of a major depressive episode is not necessary for the diagnosis of bipolar I disorder. Bipolar I disorder is more common in women than men and usually presents in young adulthood. There is a genetic component to the disease, and trauma or stress may trigger a manic episode. Most patients with bipolar I disorder also have at least one other comorbid psychiatric illness, such as anxiety disorder, substance abuse disorder, or personality disorder. Treatment of bipolar I involves hospital admission if the patient presents in overt psychosis or with marked disruption of daily activities. Pharmacotherapy during mania includes both a mood stabilizer, such as lithium or valproate, and an antipsychotic medication. Patients with bipolar I disorder who present during a depressive episode, such as this patient, can be placed on a combination of antipsychotic and antidepressant, such as olanzapine and fluoxetine. Antidepressants alone should be avoided, as they can provoke mania. Patients with bipolar I disorder are at increased risk for suicide during depressive episodes and have an increased incidence of suicide completion as compared to those with major depressive disorder. Psychotherapy is beneficial in the treatment of bipolar I disorder and is most efficacious when combined with pharmacotherapy.
A 24-year-old man presents with otalgia, pruritus, and purulent drainage from his right ear. He just returned from a swimming camp. Physical examination reveals erythema and edema of the external auditory canal accompanied by purulent drainage. Manipulation of the auricle elicits pain. What is the first step in the pathogenesis of the suspected diagnosis? ABreakdown of the skin-cerumen barrier BElevation of pH of the ear canal CImpairment of epithelial migration DNarrowing of the isthmus
A Breakdown of the skin-cerumen barrier Explanation: Otitis externa is an infection of the external auditory canal by gram-negative rods (such as Pseudomonas aeruginosa, Proteus) or fungi (such as Aspergillus). There is often a history of recent swimming or mechanical trauma. Complications of otitis externa in diabetic or immunocompromised individuals include malignant otitis externa, osteomyelitis of the skull base, meningitis, and brain abscess. The breakdown of the skin-cerumen barrier is the first step in the pathogenesis of otitis externa. Patients with otitis externa usually present with otalgia, pruritus, and purulent drainage from the affected ear. Physical examination reveals erythema and edema of the ear canal, often with purulent drainage. Manipulation of the auricle or the pinna precipitates pain. Diagnosis of otitis externa is made clinically. Treatment involves protecting the affected ear from further exposure to moisture and avoidance of additional mechanical trauma. A mixture of isopropyl alcohol and white vinegar is effective as a drying agent. When infected, otitis externa can be treated with an otic solution of aminoglycoside (neomycin-polymyxin B) or a fluoroquinolone (ciprofloxacin) with or without hydrocortisone. When cellulitis of the preauricular tissue has developed, systemic ciprofloxacin is usually effective. Aminoglycosides should be avoided in cases where the integrity of the tympanic membrane is unknown. Patients with severe disease may require placement of a wick if swelling prevents the access of topical antibiotics into the ear canal.
A 55-year-old man presents to your office with concerns about his blood pressure. He has been monitoring his blood pressure at home, and readings over the past month have all been greater than 140/90 mm Hg. Vital signs at the visit today include a blood pressure of 152/93 mm Hg and pulse of 86 beats per minute. Which of the following is the most appropriate initial therapy? AChlorthalidone BDoxazosin CHydralazine DMetoprolol
A Chlorthalidone Explanation: Hypertension is one of the most common medical conditions in the United States, with approximately 86 million adults being affected. Primary hypertension makes up the majority of cases and is caused by environmental or genetic causes. Secondary hypertension is less common and has many etiologies, including vascular, endocrine, and renal diseases. Hypertension is defined by a systolic blood pressure of ≥ 130 mm Hg or a diastolic blood pressure of ≥ 80 mm Hg or if the patient is currently taking antihypertensive medication. Hypertension occurs equally in men and women between ages 45 and 65 years. More men have hypertension younger than 45 years, and more women have hypertension over 65 years. Diagnosis is made with confirmation of an elevated blood pressure reading greater than ≥ 130 mm Hg systolic or ≥ 80 mm Hg diastolic on at least three occasions. Once diagnosed, a thorough history should be obtained to assess for any end-organ damage, determine the cardiovascular risk status, and rule out any secondary causes of hypertension. Patients may be asymptomatic and present with undiagnosed and uncontrolled hypertension for years. Physical exam findings include signs of hypertensive retinopathy such as cotton-wool spots, flame-shaped hemorrhages, papilledema, or arteriovenous nicking. Causes of secondary hypertension may be seen on physical exam such as renal artery bruits with renal artery stenosis, weak or absent femoral pulses with coarctation of the aorta or peripheral vascular disease, or an enlarged thyroid. All patients with hypertension should be counseled on lifestyle modifications such as sodium restriction, weight loss, decreased alcohol intake, regular physical exercise, and smoking cessation. First-line pharmacotherapy in general population who is not Black is with an agent from one of four classes: thiazide-like or thiazide-type diuretics, long-acting calcium channel blockers, angiotensin-converting enzyme inhibitors, or angiotensin II receptor blockers. Chlorthalidone is a thiazide-like diuretic that is recommended as initial treatment in patients with hypertension. Patients on this type of diuretic need to be monitored for metabolic complications such as glucose intolerance, hypokalemia, and hyperuricemia.
A 26-year-old woman presents to the emergency room complaining of burning with urination and exquisite vulvodynia. On exam, you find a cluster of vesicles on an erythematous base at the vaginal introitus and bimanual exam reveals an enlarged uterus. You suspect a herpes simplex viral infection. What additional historical information is most important to gather prior to your next step? ADate of last menstrual period BHistory of other sexually transmitted diseases CMedication allergies DNumber of new sexual partners
A Date of last menstrual period Explanation: Herpes simplex vaginalis is a viral infection that most commonly affects the orofacial and anogenital cutaneous areas. Risk factors for herpes simplex vaginalis include multiple sexual partners, unprotected intercourse, and contact with active herpes simplex virus lesions. The clinical presentation of herpes simplex vaginalis varies depending on whether it is an initial or recurrent outbreak. With initial outbreaks, patients typically report exquisite vulvodynia, which may worsen upon contact with clothing or urine. Other symptoms may be present, which include pruritis, dysuria, inguinal lymphadenopathy, or headache. Physical exam typically reveals a cluster of 2-3 mm clear, fluid-filled vesicles on an erythematous base. Compared to primary outbreaks, recurrent herpes simplex outbreaks tend to be less severe or more commonly asymptomatic. In instances of suspected pregnancy, it is important to determine the gestational age prior to discussion of treatment options. Therefore, in this case, asking for the date of last menstrual period is most important due to the physical finding of an enlarged uterus. Pregnant women in their first or second trimester with active herpes simplex vaginalis receive antivirals for a 7-10 day course. For patients in their third trimester of pregnancy, suppressive antiviral therapy is typically initiated at 36 weeks to prevent preterm delivery, prolonged viral shedding, and neonatal transmission, which may result in ocular and respiratory infection in the neonate. The typical antiviral of choice for herpes simplex vaginalis is acyclovir.
Which of the following therapeutics causes diminished prostate size? AFinasterideCorrect Answer BOxybutynin CTadalafil DTerazosin
A Finasteride Explanation: Benign prostatic hyperplasia is common among men over the age of 45. The prostate gland grows with age, causing many older men to have voiding difficulties. Common presenting symptoms include dribbling, hesitancy, decreased stream, multiple nocturnal voids, straining to void, and splitting or spraying of the urinary stream. On physical exam, benign prostatic hyperplasia will present as an enlarged, smooth, nontender prostate on digital rectal exam. Urinalysis and complete blood count are not affected by benign prostatic hyperplasia, but the prostate-specific antigen level is elevated. Serum creatinine levels should be normal in patients with benign prostatic hyperplasia, and if they are elevated, a genitourinary ultrasound is warranted to rule out urinary obstruction. Treatment of benign prostatic hyperplasia begins with lifestyle modifications. Patients should be counseled to void twice (to ensure complete emptying), to avoid fluids several hours before bedtime, and avoid caffeine and alcohol. Pharmacotherapy usually begins when the symptoms are extremely bothersome to the patient. Alpha-1 adrenergic antagonists are first-line therapy. If the patient cannot tolerate the side effects of this class of medication, 5-alpha reductase inhibitors can be prescribed. This class, which includes finasteride, functions by decreasing prostate size. The effects of 5-alpha reductase inhibitors are slow-acting and may take 6 to 12 months for full effect. Combination therapy with an alpha-1 adrenergic antagonist and a 5-alpha reductase inhibitor is recommended for patients with large prostates and unsatisfactory results on either agent alone. Patients who fail to respond or whose symptoms progress despite combination therapy for one to two years should be referred for surgical consultation.
A 60-year-old woman presents with acute central vision loss bilaterally. She endorses frequent tobacco use. Family history is significant for stroke. She takes 81 mg aspirin daily. Which of the following physical examination findings suggest a diagnosis of wet macular degeneration rather than dry macular degeneration? AGrayish-green discoloration near the macula BHard drusen CIncreased retinal pigmentation DOval patches of geographic retinal atrophy
A Grayish-green discoloration near the macula Explanation: Macular degeneration is a degenerative disease that affects the macula, the central portion of the retina, resulting in central vision loss. It is the most common cause of permanent visual loss in older individuals. Central vision is needed for activities such as reading, watching television, driving, and performing other activities of daily living. The prevalence of macular degeneration progressively increases in individuals > 50 years of age. Risk factors include genetics, Caucasian race, female sex, family history, cigarette use, and regular aspirin use. Macular degeneration is classified as dry (atrophic) or wet (neovascular). Dry macular degeneration is characterized by the presence of subretinal hard drusen deposits, focal or widespread geographic atrophy of retinal pigment epithelium, pigment epithelial detachment, and subretinal pigment epithelial clumping. Wet macular degeneration is characterized by the presence of soft drusen and the growth of abnormal vessels into the subretinal space, leading to collections of subretinal fluid and blood beneath the retina, as these vessels leak. Patients with dry macular degeneration usually present with a gradual vision loss in one or both eyes. Vision loss is initially noticed as difficulty with reading, driving, or scotomas. Conversely, patients with wet macular degeneration usually present with an acute central vision loss in one or both eyes. Metamorphopsia (distortion of straight lines) is an early change in wet macular degeneration and may be evaluated using the Amsler grid. The approach to the diagnosis of macular degeneration begins with a history and physical examination to assess the rate of vision loss, whether vision loss is unilateral or bilateral, and whether vision loss is for distance or near vision. Acute vision loss may represent wet macular degeneration and require prompt evaluation by an ophthalmologist. Dilated eye examination using a slit-lamp instrument may reveal the presence of hard drusen, round or oval patches of geographic atrophy in the retina, and increased pigmentation in the setting of dry macular degeneration. Conversely, in wet macular degeneration, a slit-lamp examination may show subretinal fluid and hemorrhage, which appear as a grayish-green discoloration near the macula. Treatment of macular degeneration involves slowing its progression using antioxidants (vitamins C and E), zinc, copper, and carotenoids. Intravitreal inhibitors of vascular endothelial growth factors (ranibizumab, pegaptanib, bevacizumab, and aflibercept) may reverse choroidal neovascularization in patients with wet macular degeneration. There are no specific treatments for dry macular degeneration. Smoking may increase progression to advanced macular degeneration so these patients should be encouraged to quit smoking.
A 30-year-old man presents with recurrent nosebleeds, gum bleeding, and easy bruising. Physical exam shows ecchymoses on the bilateral upper and lower extremities but is otherwise normal. Laboratory studies show WBC 7,500 cells/microL, hemoglobin 16.5 g/dL, hematocrit 45%, platelet count 350,000/microL, prothrombin time 12 seconds (range 10 to 13.5 seconds), activated thromboplastin time 65 seconds (range 60 to 70 seconds), von Willebrand factor 40 IU/dL (range 50 to 200 IU/dL), normal ratio of von Willebrand factor activity to antigen, and 65% factor VIII (range 50% to 150%). Which of the following types of von Willebrand disease is the most likely diagnosis? AType 1 BType 2A CType 2M DType 3
A Type 1 Explanation: Von Willebrand disease is an autosomal dominant bleeding disorder characterized by a reduction or dysfunction of von Willebrand factor, a glycoprotein that binds platelets at the site of vascular injury and contributes to fibrin clot formation. Von Willebrand factor is the most common inherited bleeding disorder. There are several types of von Willebrand disease. This patient has type 1 von Willebrand disease, characterized by a reduction in the levels of von Willebrand factor, concordantly reduced von Willebrand factor activity and antigen, normal factor VIII level, and the normal ratio of von Willebrand factor activity to the antigen. Patients may be asymptomatic and only show symptoms during a bleeding challenge, such as due to trauma, surgery, or childbirth. Patients present with nosebleeds, gum bleeding, easy bruising, postpartum bleeding, and heavy menstrual bleeding. Hemarthrosis is rare but can be seen in individuals with types 2N and 3. Normal complete blood count, normal prothrombin time, and normal activated thromboplastin time are typically seen in the majority of patients with von Willebrand disease. When factor VIII is significantly reduced, activated thromboplastin time may be prolonged. Treatment is by intravenous or intranasal desmopressin. Factor VIII may be given in the setting of continued bleeding and in certain types of von Willebrand disease (2N and 3).
What lab test is required prior to inserting an intrauterine device in a woman of childbearing age?
A serum or urine human chorionic gonadotropin test.
What is dacryocystorhinostomy?
A surgical procedure that creates a passage between the lacrimal sac and the nasal cavity.
A 68-year-old man presents to the emergency department with fever and productive cough. The initial vital signs show the patient has a low-grade fever, respiratory rate of 30/min, and normal heart rate and blood pressure. He is alert and oriented and otherwise healthy. Laboratory findings are unremarkable. Chest X-ray shows an infiltrate in the right lower lobe. What is the best treatment plan based on the above information? AAdmission to the floor and start empiric antibiotics BAdmission to the floor and wait for culture results to guide antibiotics CAdmission to the intensive care unit and intravenous antibiotics DDischarge with outpatient antibiotics
AAdmission to the floor and start empiric antibiotics Explanation: Pneumonia is an infection of the pulmonary parenchyma (alveoli). It can be classified as community-acquired pneumonia or nosocomial (hospital-acquired) pneumonia. Community-acquired pneumonia is defined by pneumonia symptoms beginning outside of the hospital or within the first two days of hospital admission. Pneumonia may be caused by typical bacterial pathogens (Streptococcus pneumoniae, Haemophilus influenzae, or Moraxella catarrhalis), atypical bacterial pathogens (Mycoplasma pneumoniae, Legionella pneumophila), viruses, and less commonly fungal pathogens. The classic presentation is a rapid onset of productive cough and fever. However, fever may be absent in elderly or immunocompromised patients. Tachypnea occurs in most patients, and tachycardia is also common. On exam, findings may include crackles, rhonchi, dullness to percussion, increased tactile fremitus, and bronchophony. The presence of an infiltrate on imaging with supportive clinical findings is the gold standard for the diagnosis of pneumonia. In most cases, the diagnosis is confirmed with chest X-ray, but chest computed tomography is a more sensitive study. The treatment setting is an important part of the management of pneumonia and is determined by the severity of disease. The severity of disease is often determined by severity scores, with the two most common being the CURB-65 score and the pneumonia severity index. The pneumonia severity index is preferred because it is the most accurate, but the CURB-65 score is a reasonable alternative and is simpler. The criteria used for the CURB-65 score include confusion, blood urea nitrogen concentration > 20 mg/dL, respiratory rate > 30 breaths/min, blood pressure (systolic < 90 mm Hg or diastolic < 60 mm Hg), and age > 65 years old. Patients who have zero or one of the CURB-65 criteria are managed as an outpatient. Patients who have two of the criteria should be admitted, and patients who have at least three of the criteria should be treated in the intensive care unit. The pneumonia severity index is more complex and is based on age, comorbid illnesses, vital signs, and laboratory and radiographic findings. Patients with a pneumonia severity score of I or II are typically treated in an outpatient setting, and those with a pneumonia severity index score of at least III are admitted. Patients with either respiratory failure requiring intubation or sepsis requiring vasopressor support are admitted to the intensive care unit. Antibiotics are the main treatment for pneumonia. The antibiotics used vary according to the treatment setting. Patients managed as outpatients are usually treated with one of the following antibiotic regimens: respiratory fluoroquinolone monotherapy (levofloxacin or moxifloxacin), doxycycline monotherapy, or a combination of azithromycin and a beta-lactam antibiotic, such as amoxicillin or amoxicillin-clavulanate. Hospitalized patients with community-acquired pneumonia are typically treated with either a respiratory fluoroquinolone or a combination of a beta-lactam and macrolide. In patients who have risk factors for methicillin-resistant Staphylococcus aureus, then vancomycin or linezolid is used. In patients with risk factors for Pseudomonas aeruginosa, piperacillin-tazobactam, cefepime, ceftazidime, imipenem, or meropenem should be used.
A 72-year-old man presents to the office complaining of a sudden onset of difficulty walking and falling to the right this morning. Symptoms lasted for approximately two hours before resolving. There are no focal deficits noted on his examination in the office. A CT scan of the head is ordered. Which of the following tests should be considered if an embolic source for his symptoms is suspected? ACardiac monitoring BCarotid duplex ultrasound CConventional angiogram DMagnetic resonance imaging of the brain
ACardiac monitoring Explanation: Transient ischemic attack (TIA) is a sudden decrease in blood flow to an area of the brain resulting in a focal neurological symptom lasting less than 24 hours without evidence of tissue infarction. TIAs can be thrombotic, embolic, or involve small penetrating vessels (lacunar). Embolic syndromes typically last longer than 60 minutes and originate from extracranial arteries or the heart. Emboli from the internal carotid arteries often result in a transient monocular visual loss. Posterior circulation syndromes can be secondary to emboli from the heart, aortic arch, or dissection of the vertebral artery. Symptoms of a TIA are dependent on the area of the brain involved. These symptoms can include sensory disturbance, weakness, speech disturbance, dizziness, visual loss or disturbance, or gait ataxia. A TIA is diagnosed based on clinical features, patient history, and neuroimaging findings (or lack thereof). Patients should undergo urgent evaluation with noncontrast CT scan of the head initially. If patients do not have the ability to undergo initial testing in the first 48 hours as an outpatient, they should be admitted to the hospital. Magnetic resonance imaging of the head should be performed if symptoms are ongoing, as it is more likely to identify acute areas of infarct. Neurovascular imaging, particularly CT angiogram or magnetic resonance angiogram, is helpful to identify significant stenosis or obstructive lesions within major cranial arteries. Carotid ultrasound is helpful to identify hemodynamically-significant stenosis that may benefit from surgical intervention. Cardiac evaluation (prolonged cardiac monitoring and echocardiogram) is recommended when a cardiac embolic source is suspected. Particular lesions put patients at high risk for stroke if they have had a diagnosed TIA. These lesions include atherothrombotic stenotic lesions at the origin of the internal carotid artery that are narrowed more than 50%, intracranial atherothrombotic disease lesions at proximal basilar artery, emboli at the top of the basilar artery or middle cerebral artery stem, or dissection lesions of the internal carotid artery or at the C1-C2 level of the vertebral artery. Patients with ongoing, potentially disabling symptoms should have emergent evaluation and hospital admission to determine if they are a candidate for recombinant tissue plasminogen activator (r-tPA) alteplase or potentially mechanical thrombectomy. Secondary prevention of TIA is determined by underlying etiology. Revascularization should be considered in appropriate patients. Patients who are suspected of having embolic stroke or TIA should undergo thorough investigation to determine if they are candidates for anticoagulation. Patients are otherwise started on antiplatelet therapy, such as aspirin 81 mg or clopidogrel 75 mg in patients with underlying peripheral artery disease, and statin therapy. Management of underlying comorbidities, such as diabetes and hypertension, should also be a priority. Patients who smoke cigarettes should be counseled on tobacco cessation given the increased risk for stroke and TIA among cigarette users.
What disease mimics chronic obstructive pulmonary disease in young patients regardless of smoking history?
Alpha-1 antitrypsin deficiency.
A 42-year-old woman presents to the urgent care clinic after she was bitten by her neighbor's cat several hours ago. According to the owner, the cat is up to date on vaccinations. On physical examination, she has several puncture wounds over the dorsum of her left hand, over the thumb metacarpal, and first web space. There are some dried blood and mild erythema around the wounds, but no drainage. She is able to flex and extend her thumb and fingers but complains of pain. In addition to cleaning the wounds with an antiseptic solution, which of the following antibiotic regimens is recommended for prophylaxis? AAmoxicillin-clavulanate 875/125 mg orally twice daily for five days BDicloxacillin 500 mg orally every six hours for five days CErythromycin 500 mg orally every six hours for five days DPenicillin V 500 mg orally every six hours for five days
Amoxicillin-clavulanate 875/125 mg orally twice daily for five days Cat bite wounds tend to be deep puncture wounds, and the risk for infection is higher than with dog bites. The hand is a common location for cat bites, and there is a risk of inoculation of bacteria into the deep tissues. Bite wounds on the hands should not be closed primarily but allowed to heal by secondary intention, and antibiotic prophylaxis is indicated. Most cat bites involve mixed aerobic and anaerobic bacteria. Pasteurella multocida, a gram-negative coccobacillus, is found in the majority of cat bites. Other organisms include skin flora, such as Staphylococcus aureus and Streptococcus species, and anaerobes. Other situations where antibiotic prophylaxis is indicated include wounds undergoing primary closure, wounds on the hands, face, or genital area, wounds near a bone or joint, immunocompromised hosts including people with diabetes, deep puncture wounds, and wounds with a crush injury. Amoxicillin-clavulanate is the antibiotic of choice since it has good activity against Pasteurella multocida as well as skin flora and anaerobes. Patients who are allergic to penicillins should be treated with alternatives such as doxycycline, trimethoprim-sulfamethoxazole, or a fluoroquinolone in combination with clindamycin or metronidazole to cover anaerobes. In addition to antibiotics, tetanus prophylaxis should be administered if the patient is not up to date or her vaccination status is unknown. The patient should be treated with antibiotics for three to five days and followed closely for signs and symptoms of infection.
Which of the following patients is most likely to develop medication-induced esophagitis? A24-year-old student taking omeprazole with an extra-large coffee and an energy drink B46-year-old man taking ibuprofen ten minutes before going to bed C52-year-old woman taking venlafaxine with 12 ounces of water every morning D69-year-old woman who sits up for 45 minutes after taking her vitamin D
B 46-year-old man taking ibuprofen ten minutes before going to bed Most medication-induced esophagitis is caused by poor medication-taking hygiene. The 46-year-old man taking his medications ten minutes before going to bed is most likely to develop esophagitis because he is taking a nonsteroidal anti-inflammatory a few minutes before lying in a recumbent position and sleeping. This scenario represents the most common behavior leading to medication-induced esophagitis. Lying down shortly after taking medications is an independent risk factor for medication-induced esophagitis. In addition, decreased saliva production and swallowing during sleep is an additional risk factor. All medication should be taken with at least eight ounces of water and food if possible. Patients should be instructed to remain upright (either sitting or standing) for at least 30 minutes after taking their medications. Advanced age is a nonmodifiable risk factor, likely due to decreased saliva production, increased medication use, and increased rates of esophageal motility disorders, that may predispose a patient to develop this condition. Medication drug classes that are more likely to cause medication-induced esophagitis are antibiotics (especially tetracycline, doxycycline, and clindamycin), nonsteroidal anti-inflammatories including aspirin, bisphosphonates, iron preparations, and potassium chloride. Most patients with medication-induced esophagitis lack a history of esophageal or stomach conditions. The classic presentation includes acute onset of odynophagia, retrosternal chest pain, and dysphagia. Patients may report the odynophagia is so severe that they cannot swallow their own saliva. Typically, a thorough history of how the medication is being taken will reveal the reason for the complaint. While diagnosis is often made based on history, an esophagogastroduodenoscopy should be performed on people with very severe symptoms, unusual symptoms (e.g., weight loss, hematemesis, abdominal pain), or with symptoms that persist for more than one week after the culprit medication and behavior were discontinued. Culprit medications should be discontinued if possible. If the medication cannot be stopped, a liquid preparation should be given instead of the tablet or capsule form. Acid suppression with a proton pump inhibitor should be considered if it is felt that gastroesophageal reflux may be contributing. Most patients have resolution of symptoms within seven to ten days of medication discontinuation. The most common complication is stricture. Prevention is the best strategy, and patients should be instructed to maintain good medication-taking behaviors regardless of the medication.
A 58-year old man presents for a complete yearly physical examination. He has a 40-pack year history of tobacco abuse. At what age should he be screened for an abdominal aortic aneurysm? A60 years B65 years C70 years DScreening is not indicate
B 65 years Explanation: An abdominal aortic aneurysm (AAA) is the most common type of arterial aneurysm and is characterized by a full-thickness dilation of the abdominal aorta that is 50% greater than the normal size. Men are at greater risk of developing an AAA than women. Risk factors include advanced age, tobacco abuse, caucasian race, hypertension, and atherosclerosis. AAA expansion typically occurs at a rate of 0.3 to 0.4 cm per year, thus monitoring its size is important in management. The majority of patients will not experience symptoms, therefore, men who are 65 years of age or older and have smoked or had a first-degree relative with an AAA, should be screened for the condition. Occasionally, patients will complain of abdominal, back, or flank pain with a symptomatic but unruptured AAA. The classic signs and symptoms of a ruptured AAA are hypotension, back pain that is described as "tearing," and a pulsatile abdominal mass. Diagnosis is made through ultrasound for asymptomatic AAA. The modality of choice for symptomatic AAA is computed tomography (CT). Ruptured AAAs require immediate surgical repair. If a patient has an asymptomatic AAA, surgery can be considered once the AAA reaches a size where the risk of rupture exceeds the risk of surgical repair.
A 54-year-old man presents to the clinic complaining of increased daily dyspnea, causing him to stop for breath after walking only a few minutes on level ground. He has not had any exacerbations of his condition in the past year and has never been hospitalized. He currently takes daily inhaled salmeterol and inhaled albuterol as needed for relief of acute symptoms, which provided him with adequate control of symptoms in the past. Which of the following is the next step in disease management for this patient? AAdd inhaled mometasone BAdd inhaled tiotropium CReplace inhaled albuterol with inhaled ipratropium DReplace inhaled salmeterol with inhaled tiotropium
B Add inhaled tiotropium
A 5-year-old boy presents with a left earache and fever for three days. He has had a previous anaphylactic reaction to penicillin. Pneumatic otoscopy shows erythema and decreased mobility of the tympanic membrane. Which of the following is the most appropriate therapy? AAmoxicillin BAzithromycin CCefaclor DTrimethoprim-sulfamethoxazole
B Azithromycin Explanation: The patient in the vignette above has otitis media. Otitis media is a bacterial infection of the air-containing spaces of the temporal bone, usually precipitated by viral upper respiratory infections that cause eustachian tube obstruction. As a result of the obstruction, fluid and mucus accumulate in the middle ear, leading to secondary bacterial infection. The most common bacteria that cause otitis media include Streptococcus pneumoniae, Haemophilus influenzae, and Streptococcus pyogenes. Otitis media most commonly affects children. Patients usually present with otalgia, aural fullness, decreased hearing, and fever. Physical examination (otoscopy) typically shows erythema and decreased mobility of the tympanic membrane. Bullae may also be present. The tympanic membrane may bulge outward in the setting of severe otitis media and may rupture, which is typically accompanied by a decrease in pain and otorrhea. Mastoid tenderness may be present but does not solely indicate suppurative mastoiditis. Frank swelling over the mastoid bone, presence of cranial neuropathies, or abnormal neurological findings may suggest severe disease requiring urgent care. Tympanocentesis for bacterial and fungal culture may be performed in immunocompromised patients or when multiple antibiotic treatments fail to resolve the infection. Treatment of otitis media is by antibiotic therapy (amoxicillin, azithromycin, cefaclor, amoxicillin-clavulanate). Amoxicillin is considered first-line therapy for otitis media. Amoxicillin-clavulanate is preferred when there are risk factors for beta-lactam resistance (including being previously on beta-lactam in the past 30 days, concomitant purulent conjunctivitis, history of recurrent acute otitis media unresponsive to amoxicillin). For patients with immediate allergic reaction (anaphylaxis, angioedema, bronchospasm, urticaria) or delayed reaction (including Stevens-Johnson syndrome, toxic epidermal necrolysis, or hemolytic anemia) to amoxicillin, oral azithromycin, oral clarithromycin, or oral clindamycin is indicated. Oral cefdinir, oral cefpodoxime, oral cefuroxime, or parenteral ceftriaxone may be used for treating otitis media in patients with mild delayed reaction (maculopapular or morbilliform rash occurring > 6 hours). If there is no improvement in symptoms after 48 to 72 hours of therapy, two or three doses of parenteral ceftriaxone or oral levofloxacin should be initiated. Failure of the additional treatment after 48 to 72 hours should warrant referral to a pediatric otolaryngologist for tympanocentesis or pediatric infectious disease expert. Nasal decongestants may be added for symptomatic relief. Myringotomy is reserved for patients with severe otalgia, mastoiditis, or meningitis. In the setting of recurrent otitis media, long-term prophylactic antibiotic treatment using sulfamethoxazole or amoxicillin should be initiated. Failure of this regimen warrants the insertion of ventilation tubes.
A 25-year-old sexually active man presents to the clinic complaining of dysuria, fever, and rectal pain for three days. He denies penile discharge. Vital signs are within normal limits except for a temperature of 100.8°F. Urine dipstick is negative for blood, nitrites, and glucose but positive for leukocytes. Which of the following physical exam findings would be most consistent with this patient's presumptive diagnosis? A Enlarged, tender inguinal lymph nodes B Extremely tender prostate C Purulent rectal discharge D Scattered verrucous lesions on the perineum
B Extremely tender prostate Explanation: Acute bacterial prostatitis is infection of the prostate gland, usually through ascending urinary tract infection by Escherichia coli, Proteus, Klebsiella, Enterobacter, Serratia, or Pseudomonas aeruginosa. Risk factors for the development of acute bacterial prostatitis include prostate biopsy, urinary tract infection, functional or anatomical urinary tract abnormalities, and human immunodeficiency virus infection. Acute bacterial prostatitis is more common in young and middle-aged men. Signs and symptoms of acute bacterial prostatitis include fever, chills, malaise, perineal pain, dysuria, urinary frequency, hesitancy, dribbling, or retention and an exquisitely tender, firm, and edematous prostate gland. Urinalysis may reveal leukocytosis or hematuria, and complete blood count will show leukocytosis. Prostate-specific antigen may also be elevated. Digital rectal examination should be performed gently in patients with suspected prostatitis to avoid increased pain and dissemination of the infection. Prostatitis may be diagnosed clinically, but a urine culture and Gram stain should be performed to identify the causative agent and guide treatment. Empiric therapy with a fluoroquinolone or sulfamethoxazole-trimethoprim should be initiated while awaiting results of the culture. Young, sexually active men should receive antibiotic coverage for Chlamydia trachomatis and Neisseria gonorrhoeae.
A 54-year-old obese woman presents to the clinic complaining of fatigue and easy bruisability. She has a past medical history of rheumatoid arthritis, for which she takes methotrexate and prednisone, and chronic obstructive pulmonary disease, for which she takes inhaled fluticasone and tiotropium. Which of the following is most likely present on history and physical exam? ACutaneous hypertrophy BFacial plethora CHistory of constipation DHypotension
B Facial plethora Explanation: Cushing syndrome is an endocrine disorder with characteristic manifestations secondary to excessive circulating cortisol levels. The most common causes of Cushing syndrome are iatrogenic and factitious, which are mediated by supraphysiologic doses of corticosteroids. Tumors within the adrenal cortex (adrenal adenomas, adrenocortical carcinomas) may also result in excessive autonomous secretion of cortisol that is independent of adrenocorticotropic hormone. Cushing syndrome manifests with classic physical exam findings including facial plethora with a rounded "moon" face, "buffalo hump," supraclavicular fat pads, protuberant abdomen, purple striae, easy bruisability, and thin extremities. Findings on routine laboratory studies indicate impaired glucose tolerance secondary to insulin resistance along with leukocytosis, granulocytosis, and lymphopenia. The diagnosis of hypercortisolism is initially established using late-night salivary cortisol testing or overnight dexamethasone suppression testing. Confirmation of excessive cortisol production is accomplished using 24-hour urinary free cortisol and creatinine. Follow-up plasma adrenocorticotropic hormone levels can indicate the source of hypercortisolism, with values below 6 pg/mL suggesting an adrenal tumor while higher levels indicate a pituitary or ectopic adrenocorticotropic hormone-secreting tumor. A CT scan of the adrenals can detect a mass lesion. Benign adrenal adenomas and adrenocortical carcinomas should be surgically resected. Resultant adrenal insufficiency is treated with hydrocortisone replacement therapy. Patients with Cushing syndrome secondary to iatrogenic or factitious causes require taper of corticosteroids and cessation of therapy, as clinically able. Hypertension secondary to increased mineralocorticoids should be treated with spironolactone, eplerenone, or dihydropyridine calcium channel blockers (amlodipine). Patients should be treated accordingly for other cortisol-dependent comorbidities, such as osteoporosis, psychiatric disorders, diabetes mellitus, hypokalemia, muscle weakness, and infections.
Where in the nervous system do meningiomas most commonly occur? AChoroid plexus BFalx cerebri CForamen magnum DSpine
B Falx cerebri Explanation: The falx cerebri is a common location for meningiomas. Meningiomas are the most common primary brain tumor in adults. The majority of meningiomas are benign. They are more common as age progresses, with an average age of diagnosis of 65, and are more common in women. Risk factors for meningioma development include neurofibromatosis type 2, schwannomatosis, multiple endocrine neoplasia type 1, prior radiation therapy, history of breast cancer, and obesity. Meningiomas are categorized into three groups depending on their WHO morphologic criteria. A benign meningioma is WHO grade I. A meningioma with atypical or choroid cells is considered WHO grade II, and a meningioma that is malignant (anaplastic, papillary, rhabdoid) is considered WHO grade III. Meningiomas arise from the dura mater, typically at the falx cerebri, tentorium cerebelli, or venous sinuses. Less commonly, meningiomas are observed at the optic nerve sheath, choroid plexus, or spine. Typically the tumor is asymptomatic and found incidentally on imaging. If a patient is symptomatic, the location and size of the tumor determine the symptoms. Patients may present with seizures, visual changes, loss of hearing, loss of smell, change in mental status, and weakness. The tumors grow slowly over time. Meningiomas are typically noted on contrast-enhanced magnetic resonance imaging or CT scan. A meningioma appears as an extra-axial dural based mass that tapers peripherally. There may be associated hyperostosis or bony extension, which may create a palpable bulge on physical exam. Diagnosis is confirmed via pathology after tissue sample. Treatment is dependent on tumor size, symptoms, age of the patient, and comorbidities. Management may consist of observation, surgical resection, radiation, or surgery and radiation. If the meningioma is less than 2 centimeters and the patient is asymptomatic, the MRI should be repeated in three to six months. If no progression in size is noted, annual MRIs should be performed. If progression is observed, the tumor is large, or the patient is symptomatic, the patient should be referred for surgical resection (if a candidate), otherwise radiation should be considered. For WHO grade II and III meningiomas, complete surgical resection with adjuvant radiation therapy is the preferred management. Patients with cancer with metastatic disease should undergo radiation over surgical resection. Imaging should be repeated in the postoperative period to evaluate for recurrence or progression of residual disease.
A previously healthy 48-year-old woman presents to your office for her preventive health exam. Vital signs include a blood pressure reading of 142/86 mm Hg, body mass index of 31 kg/m2, and a random blood glucose level of 150 mg/dL taken with finger-stick testing. She says in the last year she had three vaginal yeast infections, and her mother has a history of diabetes. Which of the following is the most appropriate diagnostic test? AFasting C-peptide level BFasting plasma glucose level CNonfasting plasma glucose level DUrine albumin
B Fasting plasma glucose level Explanation: Goals of the preventive health exam include identifying risk factors and screening for common diseases. The incidence of type 2 diabetes mellitus is increasing in the United States and worldwide. The American Diabetes Association recommends diabetes screening for adults who have a history of overweight or obesity and have one or more risk factors for the disease, including sedentary lifestyle, a first-degree relative with diabetes mellitus, history of cardiovascular disease, clinical conditions associated with insulin resistance, such as acanthosis nigricans, and abnormal lipid levels. Screening should also be done for asymptomatic adults aged ≥ 45 years without risk factors, and patients with treated or untreated blood pressure readings > 130/80 mm Hg. Additional risk factors for the development of the disease include a history of gestational diabetes, polycystic ovary syndrome, and a history of prediabetes. The Hispanic, Black, American Indian, Asian American, and Pacific Island populations are disproportionally affected. Diagnostic criteria for type 2 diabetes mellitus include either a hemoglobin A1C level of ≥ 6.5%, a fasting plasma glucose level of ≥ 126 mg/dL, a 2-hour plasma glucose level ≥ 200 mg/dL, or a random plasma glucose ≥ 200 mg/dL in an individual with symptoms of hyperglycemia. An oral glucose tolerance test is another form of testing for diabetes mellitus but is typically reserved for use in pregnant patients due to the inconvenience of the test. Diagnosis of diabetes mellitus must be confirmed on a subsequent day by repetition of the same test.
A 22-year-old woman presents to the clinic with a mass in her right breast. She denies any pain, redness, or swelling. On exam, the mass is well-defined and mobile. What is the most likely diagnosis? ABreast cancer BFibroadenoma CInfectious mastitis DPhyllodes tumor
B Fibroadenoma Explanation: Fibroadenomas are the most common benign breast tumor. Fibroadenomas contain glandular and fibrous tissue. Multiple fibroadenomas may occur in the same breast, or they may be present bilaterally. The cause of fibroadenomas is not known, but a hormonal relationship likely exists as fibroadenomas persist during reproductive years, increase in size during pregnancy and with estrogen therapy, and typically regress after menopause. Fibroadenomas are most common in women between 15-35 years of age. Fibroadenomas usually present as well-defined and mobile masses on physical examination. Ultrasound can be used to characterize breast lesions, and a fibroadenoma typically appears as a well-defined solid mass on ultrasound. Definitive diagnosis requires a core needle biopsy or excision. However, the management of fibroadenomas varies. They can be managed with core needle biopsy or repeat ultrasound and breast examination in three to six months. If a biopsy-proven fibroadenoma is asymptomatic, then they can be left in place or removed depending on patient preference. The disadvantages of excisional surgery include scarring at the incision site, dimpling of the breast from the removal of the tumor, damage to the breast duct system, and mammographic changes due to architectural distortion. Patients who have a presumed fibroadenoma that increases in size or is symptomatic should have an excision performed to rule out malignant change and confirm the diagnosis.
An 84-year-old man with established hypertension and benign prostatic hypertrophy presents to the clinic with extreme agitation and distress for the past three days. He is batting at the air, screaming about evil birds trying to peck out his eyes. His caregiver states he has shown no signs of psychiatric illness or dementia up to now, and he is currently taking his medications as directed, which include prazosin and lisinopril. A urine dipstick in the office is positive for leukocytes, nitrites, and blood. Which of the following represents the best choice of management for this patient? AHospital admission, intravenous antibiotics, and intramuscular chlorpromazine BHospital admission, intravenous antibiotics, and oral risperdone COral ciprofloxacin and increased fluids administered at home DOral ciprofloxacin, discontinuation of lisinopril, and return visit in two days
B Hospital admission, intravenous antibiotics, and oral risperdone Psychosis is defined as loss of contact with reality that may result in hallucinations, delusions, agitation, aggression, or disorganized thoughts. Hallucinations refer to sensory disturbances that are often distressing and not actually physically present. Delusions are strongly held false beliefs. Disorganized thought presents as defective speech patterns. Paucity of speech, abrupt interruption of speech, loose association of words or ideas, clanging speech, word salad, nonsensical words, and perseveration are examples of disordered speech patterns. Agitation presents as heightened emotions, anxiety, or psychomotor agitation. Psychosis may be the result of medication, illicit drug use, infection, psychological disorder, or chronic disease. When a patient presents with acute psychosis, treatment with antipsychotics should be initiated, regardless of the etiology of the psychotic episode. Treatment of underlying medical or psychological disorders can occur at the same time as the administration of antipsychotics. Discontinuation of offending agents is necessary, as is treatment of any underlying infection. Highly agitated patients with psychosis should be hospitalized. Many antipsychotic medications exist, and the choice of agent is often dependent on side effect profile and concurrent disease processes. This patient has benign prostatic hypertrophy and would be at increased risk of urinary retention if chlorpromazine was administered. Haloperidol has less anticholinergic side effects and would be a better choice however this carries the risk of parkinsonism in the elderly. Antipsychotics as a whole should be used with caution in the elderly, due to the potential for sedation, orthostatic hypotension, and QT prolongation. Oral risperdone is a reasonable alternative with relatively low side effects. Elderly patients with dementia who present with psychosis are at increased risk of death when antipsychotics are administered. If psychosis is related to a reversible cause, such as infection, the antipsychotic medication should be continued for two weeks after resolution of symptoms and then slowly tapered. For patients with schizophrenia or chronic psychological disorders, long-term treatment with antipsychotics is warranted.
A 25-year-old woman presents to the clinic with yellow vaginal discharge and dyspareunia for a week. On exam, yellow frothy discharge and a strawberry cervix is seen. What is the most sensitive diagnostic test for the most likely diagnosis? ACervical cytology BNucleic acid amplification test CVaginal pH testing DWet mount
B Nucleic acid amplification test Explanation: Trichomoniasis is a genitourinary infection caused by the protozoa Trichomonas vaginalis. The clinical manifestations vary. It may present as an acute and severe inflammatory disease or women may be asymptomatic carriers for as long as three months. Asymptomatic carriers may develop symptoms at any point. The most common manifestations include yellow-green, purulent, frothy, and malodorous vaginal discharge associated with burning, pruritus, dysuria, and dyspareunia. Common exam findings include erythema of the vulva or vagina. Strawberry cervix due to punctate hemorrhages on the cervix is seen in some cases. However, there are no symptoms or signs specific enough to distinguish trichomoniasis from other causes of vaginal discharge. Therefore, the diagnosis of trichomoniasis is confirmed with laboratory testing. There are a variety of laboratory methods that can be used to confirm the diagnosis. These include wet mount (microscopy), culture, rapid antigen detection tests, and nucleic acid amplification testing, which is performed by polymerase chain reaction. Culture results take three to seven days, but the sensitivity is about 96%. Rapid antigen detection tests are resulted within an hour and have a sensitivity of about 90%. Nucleic acid amplification testing is the most accurate diagnostic method. These tests each have high specificity. Treatment is recommended in asymptomatic and symptomatic women with trichomonas vaginalis. Nonpregnant women should be treated with a single oral dose of metronidazole or tinidazole. The sexual partners of patients with trichomoniasis should also be treated. Patients should be educated to avoid intercourse until all their partners have been treated and they are asymptomatic. Retesting is recommended within three months of completing treatment.
A 54-year-old woman presents to your office with a bump over her right wrist, which she first noticed about six months ago. It is not painful and has not changed in size. On examination, she has a round, mobile, nontender, and well-circumscribed mass about 2 cm in diameter over the dorsum of her wrist. The mass transilluminates with a flashlight. She has full range of motion of her wrist. X-rays of the wrist are normal. Which of the following is the most appropriate next step in management? AInjection of the mass with a glucocorticoid BObservation and reassurance CRupture of the mass by hitting it with a heavy object DSurgical excision of the mass
B Observation and reassurance Explanation: Ganglion cysts are fluid-filled masses that arise from joints or tendon sheaths. The cause is unknown, but they are thought to be due to mucoid degeneration and contain a mucinous, gelatinous fluid. They are the most common soft tissue masses of the hand and wrist. The locations most often seen are the dorsum of the wrist at the scapholunate joint, followed by the volar radial aspect of the wrist, and the volar base of a finger. They are usually asymptomatic but may cause symptoms if they compress nerves or blood vessels. Physical examination reveals a well-circumscribed, mobile mass, and transillumination confirms the mass is cystic rather than solid. Ultrasound can also be used to differentiate a cyst from a solid mass. X-rays are usually not necessary but may be used to rule out other conditions. Advanced imaging is rarely required but may be useful if an occult ganglion is suspected. The clinician should reassure the patient the ganglion cyst is a common, benign condition and that over half will resolve spontaneously. Aspiration of the cyst may be indicated for symptomatic patients. Volar cysts are closely associated with the radial artery, so care should be taken to avoid injuring the artery during aspiration. The presence of mucinous fluid is diagnostic. The recurrence rate after aspiration is about 50%.
A 65-year-old man with a history of diabetes and uncontrolled hypertension presents to the urgent care clinic with a complaint of vision loss in his left eye. He states that he had an episode earlier in the day of blindness in that eye, but the vision returned quickly and it was not painful, so he was not worried. This time, the vision loss has lasted about an hour. He denies any eye pain. On exam, the patient can only see shadows of fingers from the left eye. A carotid bruit is audible over the left carotid artery. Fundoscopic exam shows whitening of the retina with a red area in the fovea. Given the patient history and physical exam, which of the following is the most likely condition? AOptic neuritis BRetinal artery occlusion CRetinal detachment DRetinal vein occlusion
B Retinal artery occlusion Explanation: Retinal artery occlusion is a condition most commonly caused by atherosclerotic disease affecting the carotid artery on the ipsilateral side. As with other vascular conditions, the risk factors include hypertension, smoking, diabetes, and hypercoagulability. The average age for this occurrence is between 60 and 65 years old and is more common in men. It presents as a severe, acute, painless, monocular loss of vision that may be preceded by flashing lights or transient blindness. The retinal occlusion can affect either the central or the branch retinal arteries. In the case of a central occlusion, the vision loss is more severe. Branch occlusion may be restricted to only a portion of the visual field. On fundoscopic exam, the ischemia caused by the occlusion causes the retina to whiten or become translucent, leaving a cherry-red spot on the macula. Retinal emboli may also be visible. Retinal artery occlusion is an ocular emergency with a poor prognosis especially if left untreated for more than 24 hours. Treatments are most effective within the first six hours. The condition is considered similar to a stroke, and revascularization with either thrombolytics or surgical techniques should be considered. Other treatment options are available for those who are not candidates for these treatments. Once a retinal artery occlusion has occurred, patients are at a higher risk for both cardiovascular and cerebrovascular events. Long-term treatment may include antiplatelet therapy and chronic anticoagulation.
A 45-year-old man is known to have human immunodeficiency viral infection. His last CD4 count was 100 cells/mcL, and he was started on antiretroviral therapy at that time. His current CD4 count is 150 cells/mcL. He presents to his primary care physician asking about preventive immunizations. Which of the following vaccinations is contraindicated in this patient? AInactivated influenza vaccine BMeasles, mumps, and rubella vaccine CMeningococcal vaccination DPneumococcal vaccination
BMeasles, mumps, and rubella vaccine Explanation: In 2013, the Infectious Diseases Society of America published guidelines for vaccination of immunocompromised individuals, including those who are infected with HIV. It was found that vaccines tend to be less immunogenic and antibody response is more short-lived in individuals who are HIV-infected compared to the general population. Additionally, certain vaccines have better efficacy when administered early, preferably prior to a decline in CD4 count. Inactivated vaccines are generally safe and acceptable in HIV-infected individuals, while live vaccines should not be used in HIV-infected individuals whose CD4 count is less than 200 cells/mcL. A CD4 count less than 200 cells/mcL is evidence of severe immunosuppression, and live vaccines (measles, mumps, rubella, and varicella) can cause severe illness in the patient in the vignette above given his immunocompromised state. Hence, the measles, mumps, and rubella vaccine is not recommended in this patient. However, postexposure prophylaxis with varicella zoster immune globulin following a varicella zoster infection may be sought in an HIV-infected individual who does not have immunity through natural infection or immunization. Inactivated vaccines that are recommended for HIV-infected individuals include inactivated influenza, tetanus/diphtheria, human papillomavirus, pneumonia, and meningitis.
What peripheral blood smear finding is characteristic of glucose-6-phosphate dehydrogenase deficiency?
Bite cells (Heinz bodies).
What distinguishes schizoaffective disorder from bipolar I disorder?
Both disorders may present with psychotic symptoms, but the psychosis will only be present during episodes of mania or depression in bipolar I disorder and will persist in the absence of a mood disturbance in schizoaffective disorder.
A 74-year-old man presents to the clinic for a routine physical examination. He has no specific concerns. Review of his previous visits reveals a 12% loss of body weight in the past 3 months. Physical examination demonstrates the cutaneous findings shown above in addition to a firm, round, mobile enlarged cervical lymph node. Complete blood cell count results indicate an absolute lymphocyte count of 50,789/microL. Which of the following is the most likely diagnosis? AAcute lymphocytic leukemia BAcute myelogenous leukemia CChronic lymphocytic leukemia DChronic myelogenous leukemia
C Chronic lymphocytic leukemia Explanation: Chronic lymphocytic leukemia is a neoplastic disease that is characterized by progressive accumulation of monoclonal B lymphocytes. It is the most common adult leukemia in resource-rich countries and represents more than one-quarter of all leukemias in the United States. It is more common in male, older, and White patients. First-degree relatives of patients with chronic lymphocytic leukemia may be at increased risk for the disease, in addition to other lymphoid, hematologic, and solid tumors. Patients with chronic lymphocytic leukemia are often asymptomatic but may report painless lymph node swelling that waxes and wanes, unintentional weight loss (≥ 10% body weight in 6 months), fevers higher than 100.5°F for at least 2 weeks without evidence of infection, night sweats, and extreme fatigue. Lymphadenopathy is the most common abnormal finding on physical exam, with the most frequently affected sites being cervical, supraclavicular, and axillary nodes. Splenomegaly and hepatomegaly may be noted on palpation of the abdomen. Leukemia cutis skin lesions often involve the face and have a varied appearance with single or multiple red-brown, violaceous, or hemorrhagic papules, nodules, or plaques. Confirmation of infiltration by chronic lymphocytic leukemia involves skin biopsy. Unlike other lymphomas, gastrointestinal mucosal involvement is rare in patients with chronic lymphocytic leukemia. Laboratory evaluation demonstrates lymphocytosis, with absolute blood lymphocyte > 5,000/microL. Neutropenia, anemia, and thrombocytopenia may also be present. Hypogammaglobulinemia is present at initial diagnosis and, in combination with neutropenia, predisposes a patient to major bacterial infection. Peripheral smear demonstrates lymphocytosis with "smudge" cells. Flow cytometry reveals a single clone of abnormal circulating B lymphocytes. Bone marrow aspirate and biopsy are not typically necessary to diagnose chronic lymphocytic leukemia. Chronic lymphocytic leukemia is staged according to either the Rai or Binet staging systems, which take into consideration affected lymph nodes and organs and the presence of hematologic abnormalities, such as anemia and thrombocytopenia. Patients with early stage, asymptomatic disease are best treated with observation. Advanced or symptomatic disease is treated with chemotherapeutic agents, such as cyclophosphamide and rituximab. Certain patient populations may be alternatively treated with ibrutinib, a tyrosine kinase inhibitor, or fludarabine, a purine analog antimetabolite. Survival after diagnosis varies from 2 to 20 years, with a median of 10 years.
A 60-year-old man with a history of atherosclerotic heart disease presents with retrosternal chest pain for 20 minutes. Pain is rated at 10/10, and he states it radiates up the left side of his neck. He is a current frequent tobacco user. On evaluation, he is noted to be anxious and diaphoretic. ECG shows ST elevation in leads II, III, and AVF. Which of the following cardiac biomarkers is useful for evaluating early reinfarction in this patient? AB-type natriuretic peptide BC-terminal provasopressin CCreatinine kinase-MB DTroponin I
C Creatinine kinase-MB Explanation: Coronary artery disease is an impairment of blood flow through the coronary arteries, most commonly caused by atherosclerosis. Coronary artery disease is the leading cause of death in industrialized countries. Risk factors include dyslipidemia, diabetes mellitus, smoking, obesity, and chronic inflammatory states. Clinical manifestations of coronary artery disease include angina pectoris, unstable angina, myocardial infarction, silent myocardial ischemia, and sudden cardiac death. Angina pectoris is precordial chest pain that is aggravated by stress or exertion and relieved by rest or nitroglycerin. Unstable angina results from an acute coronary artery obstruction without infarction and is described as rest angina, new-onset angina, or previous angina that is more severe, lasts longer, or occurs more frequently. Myocardial infarction (MI) is necrosis of the myocardium that results from an acute and complete occlusion of a coronary artery. Patients with MI typically present with substernal chest pain that may or may not radiate to the left arm or left neck, diaphoresis, nausea, vomiting, dyspnea, and fatigue. MI is divided into ST elevation myocardial infarction (STEMI) and non-ST elevation myocardial infarction (NSTEMI). Diagnosis is by serial ECG, serial cardiac markers, immediate coronary angiography for patients with STEMI, and delayed coronary angiography for patients with NSTEMI. ECG should be done within 10 minutes of presentation and may show changes indicative of ischemia (unstable angina), ST depression (NSTEMI), and hyperacute T waves, T wave inversion, and ST-segment elevation with reciprocal depression (STEMI). Cardiac markers should be ordered and include B-type natriuretic peptide, creatinine kinase-MB, troponins I and T, and C-terminal provasopressin. Serum levels of creatinine kinase-MB typically rise in 3 hours, peak within 24 hours, and normalize within 48-72 hours. Thus, it is a more helpful lab when evaluating for reinfarction. Troponins should be ordered initially whenever MI is suspected.
A 23-year-old man with no clinically significant history presents with chest pain for two days. He had been coughing for three weeks before he developed chest pain. He denies any recent travels or surgery. Pain is substernal, nonradiating, and worse with coughing or sneezing. Physical examination is normal. Pain is reproducible with palpation of the chest wall around the rib cage. Which of the following is the most appropriate next step in diagnosis? AChest radiograph BD-dimer CElectrocardiogram DTroponin assay
C Electrocardiogram Explanation: Chest pain is a common symptom that can occur as a result of cardiopulmonary, pleural, musculoskeletal, esophageal, and gastrointestinal disorders, cocaine use, Herpes zoster, or anxiety states. The approach to chest pain as a presenting complaint involves a comprehensive history and physical examination, including the pain onset, character, location/size, duration, exacerbations, accompanying shortness of breath, age, comorbidities, vital signs, ECG, laboratory studies, and imaging. Life-threatening differential diagnoses, including acute coronary syndrome, aortic dissection, pericarditis, vasospastic angina, pulmonary embolism, pneumonia, and esophageal perforation must be considered promptly. Myocardial ischemia typically presents as a dull, aching sensation of "pressure," "tightness," or "squeezing" retrosternal pain, lasting 5-20 minutes, radiating to the throat, upper jaw, shoulders, upper abdomen, or back. Atypical presentations are seen in older individuals, females, and diabetics. Pericarditis produces pleuritic pain that is worse supine than upright. Abrupt onset of tearing pain radiating to the back is seen in aortic dissection. Individuals with pulmonary embolism often present with chest pain but will also have one or more of the Virchow triad (stasis, hypercoagulable state, or vascular endothelial injury). Ruptured esophagus occurs iatrogenically or secondary to vomiting, and is usually accompanied by subcutaneous emphysema. A reproducible pain upon palpation is likely due to a musculoskeletal etiology. ECG is warranted initially in all patients presenting with chest pain to evaluate for acute coronary syndrome, pericarditis, or pulmonary embolism. With normal or nonspecific ECG findings and normal physical examination, the patient in the vignette can be safely removed from cardiac monitoring. This decision rule has a 100% sensitivity. Chest radiograph should be ordered next in this patient.
A 43-year-old woman presents to the clinic complaining of three days of cough with "rust colored" sputum production. Vitals reveal HR 102 bpm, RR 17 breaths/min, T 101.8°F, BP 127/81 mm Hg, and SpO2 96% on room air. Which of the following findings is most likely present on physical exam? ADiffuse wheezes BExpiratory crackles CIncreased tactile fremitus DVesicular breath sounds
C Increased tactile fremitus
A 36-year-old woman presents to your clinic to discuss contraceptive options. She has three children, ages 7, 9, and 12. With all three children, she had normal pregnancies and uncomplicated vaginal births. Before her first child, she had one spontaneous abortion at nine weeks. Her menses are irregular, occurring every 17 to 34 days, and she experiences moderate-to-severe abdominal cramping on cycle days one through four. She is not interested in childbearing at this time, but does not desire permanent sterilization. She is interested in a hormonal intrauterine device. When discussing this form of contraception, what common side effect of hormonal intrauterine devices would you discuss? AAnovulation BDysmenorrhea CMetrorrhagia DTransient bone marrow density loss
C Metrorrhagia Explanation: Intrauterine devices are long-acting reversible contraceptives that range in efficacy from 3-10 years. Intrauterine contraceptive devices are an appropriate form of contraception for women of childbearing age who want a reversible method of contraception. There are numerous benefits of intrauterine devices. They offer highly effective pregnancy prevention, they do not require regular adherence or daily dosing, they are long-acting and rapidly reversible, they are private and do not interfere with the spontaneity of intercourse, and they avoid potentially dangerous exogenous estrogen (or in the case of copper devices, any hormone use whatsoever). Intrauterine devices function to prevent pregnancy by foreign body effect induced by the device frame and by local changes caused by the released medication (in all devices except the copper intrauterine device). As with all medications, it is important to review the unique side effect profile for intrauterine devices. While serious side effects like device perforation, uterine wall implantation, and expulsion are possible, the most common side effect of intrauterine devices is metrorrhagia. Irregular bleeding is most common in the first few months after device insertion but can persist for months or years.
Which of the following molecules facilitates the adherence and gliding motion of Mycoplasma pneumoniae along the respiratory epithelium? ABiofilm BCilia CP1 adhesin protein DToll-like receptors
C P1 adhesin protein Explanation: A common pneumonia is caused by Mycoplasma pneumoniae, a small free-living atypical bacterium that is transmitted via respiratory droplets, with an incubation period of two to three weeks. Additional clinical manifestations of M. pneumoniae include upper respiratory tract infections and acute bronchitis. M. pneumoniae is considered an atypical organism because it does not possess a cell wall. Hence, it is not visible on a Gram stain and is not susceptible to beta-lactams and other antibiotics that inhibit cell wall synthesis. M. pneumoniae possesses a specialized protein known as P1 adhesin protein that facilitates its adherence and gliding motion along the respiratory epithelium. It occurs in people living in close quarters such as college dormitories, health care facilities, and military quarters. Clinical manifestations of M. mycoplasma include a gradual onset of cough, that may be accompanied by low-grade fever, sore throat, headache, and malaise. Physical examination may reveal increase sputum production and bilateral diffuse rales. Chest radiograph typically shows interstitial infiltrates. Treatment options include macrolides (azithromycin, clarithromycin, erythromycin). Alternatively, doxycycline and tetracycline can be used in patients who are older than eight years of age.
A 55-year-old man presents to his primary care provider with a complaint of a recurring rash. Which of the following presentations would support a diagnosis of nummular dermatitis? ADry, scaling, asymptomatic lesions BErythematous, pruritic patches in areas of pitting edema CPruritic, coin-shaped lesions on extremities DSegmented hyphae on potassium hydroxide preparation
C Pruritic, coin-shaped lesions on extremities Explanation: Nummular eczema is a chronic, idiopathic condition most commonly seen in men over the age of 50. Risk factors for developing the condition include hepatitis C, isotretinoin use, and breast expanders in patients going through reconstruction. It presents as multiple pruritic, coin-shaped lesions on the extremities and only occasionally on the trunk in areas of dry skin. Initially, the lesions are dull red, exudative, and crusty and measure between 1 and 10 centimeters in diameter. The lesions become dry and scaly with an occasional central clearing giving them an annular shape. They may resolve or become dormant only to reactivate days or months later. Diagnosis is made clinically. Management includes addressing skin dryness, decreasing inflammation, and avoiding irritants. For active lesions, high potency topical corticosteroids are first-line treatment. Intralesional triamcinolone, phototherapy, and systemic immunosuppressive therapy may also be effective.
A 41-year-old woman is being evaluated for an unexpected elevation in her liver function tests. As part of the workup, a hepatitis B panel was obtained and shows a positive hepatitis B surface antibody, positive hepatitis B core antibody, and negative hepatitis B surface antigen. Which of the following represents the most accurate interpretation of her lab results? AResults of the hepatitis B envelope antigen are needed to determine if she has hepatitis BShe is immune to hepatitis B due to prior effective immunizations CShe is immune to hepatitis B due to prior natural infection DShe is not immune to hepatitis B
C She is immune to hepatitis B due to prior natural infection Explanation: This patient is immune to hepatitis B due to prior natural infection. This is determined by first looking at the result of the hepatitis B surface antigen. The surface antigen (HBsAg) will indicate if the patient currently has a hepatitis B infection. A negative surface antigen, in this case, determines that the patient is not acutely infected with hepatitis B. It is the earliest indicator of acute infection, as hepatitis B seroconversion occurs one to ten weeks after exposure to the hepatitis B virus. Second, the patient's immunity should be reviewed. A positive hepatitis B surface antibody (Anti-HBs) indicates that the patient's immune system has been exposed to the hepatitis B virus but does not indicate if it was exposed to this virus via natural infection or vaccination. To determine the reason this patient has a positive surface antibody, the hepatitis B core antibody must be reviewed. The core antibody is only created when the actual live virus was presented to the patient's immune system. Vaccination will not result in a positive core antibody. Following exposure to hepatitis B, the majority of patients have a subclinical or anicteric hepatitis. Rarely, acute hepatitis B presents as fulminant liver failure, which tends to be in patients who have other underlying liver dysfunction or infection. The two most common methods of viral transmission in the United States are sexual contact and percutaneous exposure (e.g., intravenous drug use). The incubation period ranges from one to four months. Patients who develop symptoms of acute hepatitis typically present with right upper quadrant pain, constitutional symptoms, jaundice, and nausea. In addition to a positive hepatitis B surface antigen, patients are expected to have alanine aminotransferase (ALT) > aspartate aminotransferase (AST), indicating hepatic injury. Treatment for acute hepatitis B is mainly supportive because the majority of patients will clear the infection and will not progress to chronic hepatitis B infection. Most patients who develop chronic hepatitis B will be asymptomatic for most of their lives with the exception of feeling fatigued. The physical exam may be normal unless the hepatitis has lead to liver decompensation. Decompensated hepatitis B will present similarly to any other cause of decompensated liver disease: jaundice, splenomegaly, hepatomegaly, ascites, and encephalopathy. Some patients have normal liver enzyme function tests, however, many will have chronic elevations of ALT and AST. Patients with chronic hepatitis B are at increased risk of developing hepatocellular carcinoma and should be screened annually for cirrhosis and hepatocellular carcinoma. Treatment of hepatitis B is based on antiviral therapy under the guidance of a specialist.
Which of the following organisms is the most common cause of bacterial sinusitis? AMoraxella catarrhalis BStaphylococcus aureus CStreptococcus pneumoniae DStreptococcus pyogenes
CStreptococcus pneumoniae Explanation: Bacterial sinusitis is symptomatic inflammation of the nasal cavity and paranasal sinuses by bacteria, including Streptococcus pneumoniae, Haemophilus influenzae, Staphylococcus aureus, Moraxella catarrhalis, and Streptococcus pyogenes. Of these bacteria, Streptococcus pneumoniae is the most common cause of bacterial sinusitis. Bacterial sinusitis commonly occurs as a complication of viral infection but may also be associated with rhinitis, mechanical obstruction of the nose, dental infection, and impaired mucociliary clearance. It may be uncomplicated or complicated. Uncomplicated bacterial sinusitis is an infection that is confined to the nasal cavity and paranasal sinuses while complicated bacterial sinusitis extends beyond the nasal cavity and paranasal sinuses, causing neurologic, ophthalmic, or soft tissue infections. Patients with bacterial sinusitis present with nasal congestion, purulent nasal discharge, facial pain or pressure, halitosis, fever, ear pressure, or fullness that lasts longer than 10 days. Some patients may experience a biphasic pattern illness, characterized by worsening symptoms after an initial period of improvement. A physical exam may show erythema or edema over the affected sinus, purulent drainage in the nose, pain exacerbation with sinus percussion, and maxillary tenderness. Transillumination of the frontal and maxillary sinuses may show opacity. Complications of bacterial sinusitis include preseptal cellulitis, orbital cellulitis, subperiosteal abscess, meningitis, and intracranial abscess. Imaging is not initially indicated but may be obtained in complicated cases. Treatment of bacterial sinusitis is supportive initially using analgesics, antipyretics, and intranasal irrigation. As symptoms persist, amoxicillin-clavulanate or doxycycline may be added.
Which new drug class includes monoclonal antibodies in the prevention of migraine headaches?
Calcitonin gene-related peptide antagonists.
What is pseudogout?
Calcium pyrophosphate deposition disease is caused by deposition of calcium pyrophosphate crystals in the joint, which are smaller than uric acid crystals and are weakly positively birefringent. The clinical presentation is similar to gout.
What is Mirizzi syndrome?
Common hepatic duct obstruction resulting from an impacted stone in the cystic duct or the neck of the gallbladder.
What is the treatment for giant cell arteritis?
Corticosteroids
A 60-year-old man presents to your office with a complaint of epigastric pain. He says that he typically gets the pain after eating and describes it as a burning sensation. He injured his back last year and takes ibuprofen daily for pain relief. Testing for Helicobacter pylori is negative. Which of the following is the most appropriate therapy? AAmoxicillin BBismuth subsalicylate CClarithromycin DPantoprazole
D Pantoprazole Explanation: A peptic ulcer is a defect in the wall of the stomach or duodenum that penetrates the mucosa and enters the deeper layers. The most common causes of peptic ulcer disease are Helicobacter pylori infection and the use of nonsteroidal anti-inflammatory medications (NSAIDs). Other etiologies include lifestyle factors such as tobacco and alcohol use, physiologic stress, genetic factors, and hypersecretory states. Normally there is a physiologic balance between the gastroduodenal mucosal defense and secretion of gastric acid. Peptic ulcers occur when that balance is disrupted and mucosal injury occurs. Peptic ulcer disease affects approximately 4.5 million people in the United States each year and has similar rates of occurrence in men and women. A thorough medical history is essential in making the diagnosis of peptic ulcer disease with specific attention paid to the use of NSAIDs, history of Helicobacter pylori infection, and smoking. Epigastric pain is the most common symptom of peptic ulcer disease, regardless of the location of the ulcer. The pain is described as a burning or gnawing sensation shortly after eating for a gastric ulcer and 2-3 hours later for a duodenal ulcer. Individuals with duodenal ulcers will feel pain relief with antacids or food, but these provide minimal relief in patients with gastric ulcers. Other symptoms include dyspepsia, chest discomfort, heartburn, and hematemesis or melena. Alarm features for patients with peptic ulcer disease include anemia, bleeding, unintentional weight loss, recurrent vomiting, early satiety, progressive dysphagia, and a family history of gastrointestinal cancer. Physical exam findings may be normal or nonspecific such as a mild epigastric tenderness or guaiac-positive stool testing. Testing for Helicobacter pylori infection should be done in all patients with peptic ulcer disease. Diagnosis of peptic ulcer disease is confirmed with radiography and endoscopy. Management begins with determination of the need for treatment of Helicobacter pylori infection. Patients with Helicobacter pylori infection should complete treatment and then undergo repeat testing four weeks after completion of therapy to confirm eradication. Patients should be counseled to stop all NSAIDs, quit smoking, and limit alcohol intake to one drink per day or less. All patients with peptic ulcers should receive antisecretory therapy to help heal the ulcers. Individuals with an NSAID-induced ulcer should receive eight weeks of therapy with a proton pump inhibitor such as pantoprazole, omeprazole, or esomeprazole. High-risk patients are those with recurrent, complicated, or giant ulcers, and these individuals require one year of antisecretory therapy with either proton pump inhibitors or H2 blockers such as ranitidine.
A 58-year-old man presents to the clinic complaining of headache, chills, and nausea that began seven hours after he ate a dish containing fava beans. He has a medical history of cirrhosis secondary to nonalcoholic steatohepatitis. Which of the following is the best indication of hemolytic anemia in this patient? ADecreased haptoglobin BElevated direct bilirubin CMacroovalocytes on peripheral blood smear DPositive urine hemosiderin
D Positive urine hemosiderin Explanation: Hemolytic anemias are a group of blood disorders characterized by reduced red blood cell survival. Normal red blood cells are produced by the bone marrow with an average lifespan of 120 days. Anemia secondary to these conditions only results when there is significantly reduced red cell survival rates or when the bone marrow is unable to compensate for increased red blood cell turnover. Hemolytic anemias manifest secondary to intrinsic defects of red blood cells or to an extrinsic factor. Intrinsic defects include membrane defects (e.g., hereditary spherocytosis), glycolytic defects (e.g., pyruvate kinase deficiency), oxidation vulnerability (e.g., glucose-6-phosphate dehydrogenase deficiency), and hemoglobinopathies (e.g., sickle cell syndromes, thalassemia). Extrinsic effects are more often immune-mediated, due to infection with the pathogens Plasmodium, Clostridium, or Borrelia or secondary to hypersplenism or burns. Microangiopathic anemias, another extrinsic type of hemolytic anemia, include thrombotic thrombocytopenic purpura, hemolytic-uremic syndrome, disseminated intravascular coagulation, valve hemolysis, metastatic adenocarcinoma, vasculitis, and copper overload. Hemolytic anemias share characteristic laboratory findings. Complete blood count results reveal decreased hemoglobin or hematocrit with normal white blood cell and platelet count. Macrocytosis is common, but due to a high level of anisocytosis, the mean corpuscular volume may be low or normal. The peripheral blood smear is an essential component of diagnostic evaluation and can provide clues as to the underlying disorder. Some examples of findings on peripheral blood smear include target cells in hemoglobinopathies, sickle cells in sickle cell syndromes, and teardrop-shaped cells in thalassemic disorders. Schistocytes are also a common finding on peripheral blood smear in various hemolytic anemias. Reticulocyte count and percentage indicate the rate of production of red blood cells in the bone marrow. A normal reticulocyte count is between 25,000 to 75,000/microL while a normal reticulocyte percentage is 1 to 2%. Due to the increased rate of cell turnover secondary to hemolysis, the reticulocyte count in hemolytic anemia is elevated with a corresponding reticulocyte percentage of 4-5% or higher. Lactate dehydrogenase and total and indirect bilirubin are often elevated. Once the renal tubular cells have reached capacity for reabsorption of hemoglobin, hemoglobinuria will be present. Prior to this point, intravascular hemolysis can be detected by a positive urine hemosiderin. Haptoglobin is a normal plasma protein that binds and clears free hemoglobin that has been released into plasma. Due to the increased demand proportional to increased red blood cell hemolysis, haptoglobin levels are decreased with these anemias. This protein level is affected by a variety of factors (e.g., inflammation) and can be falsely low in cases of end-stage liver disease, congenital ahaptoglobinemia, and abdominal trauma. Further evaluation specific to the suspected underlying hematologic disorder that caused the hemolytic anemia should be implemented. The hemolytic anemia should be treated according to the underlying condition.
A 25-year-old man who is a construction worker presents with a growth on his right eye that he noticed four months ago. The growth has progressively become larger to the extent that his vision is now impaired. Physical examination reveals a triangular growth of the conjunctiva on the nasal side of the cornea. Which of the following is the most likely diagnosis? AChalazion BHordeolum CPinguecula DPterygium
D Pterygium Explanation: Pterygium is a fleshy, triangular growth of the bulbar conjunctiva that occurs on the nasal side of the cornea and is usually caused by prolonged exposure to wind, sun, sand, and dust. It is usually bilateral and may become inflamed. Pterygium may grow and cause vision impairment, eye irritation, and redness. It is most commonly seen in men, especially in those that work outdoors, including roofers, construction workers, and landscapers. Diagnosis is usually made clinically. No treatment is indicated for pterygium, but artificial tears and lubricants may be used for symptomatic relief of eye irritation. Surgical resection of pterygium is indicated when vision impairment, severe ocular irritation, or marked induced astigmatism occurs. Recurrence of pterygium is common and is usually more severe than the primary lesion.
A 70-year-old man presents to your office with a complaint of thickened toenails. Physical exam of the feet reveals dystrophic toenails on all digits that are dark yellow in color. He says that they are starting to hurt when he wears tight-fitting shoes. Which of the following is the most appropriate oral therapy? AFluconazole BItraconazole CKetoconazole DTerbinafine
D Terbinafine Explanation: Onychomycosis is a fungal infection of the fingernails or toenails that can cause pain and disfigurement of the nail. The infection can affect any part of the nail, including the matrix, nail plate, or bed. Onychomycosis is caused by three types of fungi: yeasts, dermatophytes, and nondermatophyte molds, with dermatophytes being the most common cause. Men are affected more than women, and older individuals are affected more than children with up to 90% of the elderly having onychomycosis. The pathogenesis of the infection depends on the type of fungus involved. The fungus may spread from the plantar skin to invade the nail bed, or the nail may be directly infected. Patients initially present with complaints of the nail appearance and no other symptoms. In more severe cases, they may experience interference with walking, standing, or exercise, paresthesias, discomfort, and pain. Because of the cosmetic appearance, loss of self-esteem or social interactions may occur. Laboratory diagnosis should be made prior to initiating therapy, as the appearance of onychomycosis can mimic other nail disorders. Direct microscopy using potassium hydroxide preparation is a screening test to rule out the presence of fungus. Fungal culture is used to identify the species of fungal organism involved. Treatment is not required for all patients. Treatment for onychomycosis should be initiated in patients with a history of cellulitis, diabetes mellitus, complaints of pain, or request for cosmetic improvement and in immunocompromised patients. Treatment options include oral and topical antifungal agents, photodynamic therapy, laser therapy, and surgery. Topical and systemic medications are most commonly used with the selection of an agent based on a number of factors, including type of organism, severity of disease, side effects, ability to monitor side effects, cost, and availability. Topical agents have less risk for serious side effects and interactions with other medications but generally require longer courses of treatment and may not be as effective in extensive or severe disease. Oral terbinafine is the first-line treatment option for mild to moderate onychomycosis in patients without contraindications to its use. Individuals with active or chronic liver disease should avoid using oral terbinafine. The course of treatment is 6 weeks for fingernails and 12 weeks for toenails. Side effects of terbinafine include liver enzyme abnormalities, headache, dermatitis, gastrointestinal upset, and taste disturbances. Monitoring of transaminase levels should be done at baseline and then at six weeks.
A 30-year-old man presents with bloody diarrhea and mild, crampy abdominal pain for five months. He reports associated tenesmus and mucus in the stool. Stool cultures were negative. Serum testing shows mild normocytic anemia and an elevated sedimentation rate. What is the most likely diagnosis? AColorectal cancer BCrohn disease CIrritable bowel syndrome DUlcerative colitis
D Ulcerative colitis Explanation: Ulcerative colitis is a condition causing inflammation to the mucosal layer of the colon. It typically does not involve the small intestine. The classic presentation of ulcerative colitis is bloody diarrhea with tenesmus, commonly reported as frequent small volume stools with an incomplete sense of evacuation due to rectal inflammation. Abdominal cramping and mucus in the stool is also frequently reported. Ulcerative colitis is more common in men who are of a young age. Patients often report their symptoms have been gradual and progressive over the course of a few weeks, eventually prompting them to seek care. Severe disease manifestations include weight loss, blood loss anemia, fatigue, fulminant colitis, and toxic megacolon with subsequent perforation. Extraintestinal manifestations, such as arthritis, ocular complications, arterial or venous thrombosis, and erythema nodosum have been observed primarily following an acute exacerbation of gastrointestinal symptoms. These patients also have an increased risk of developing primary sclerosing cholangitis and colorectal carcinoma. Screening for colorectal cancer is dependent on when a patient develops their first episode of pancolitis, as screening with a colonoscopy should begin eight to ten years following this diagnosis. Typical lab findings are normal to anemic complete blood count, elevated sedimentation rate, low albumin, or electrolyte imbalances due to diarrhea. Imaging is not required to make the diagnosis but is often ordered to rule out other causes of abdominal pain and diarrhea. CT findings frequently show nonspecific colitis (e.g., bowel wall thickening). The workup of ulcerative colitis also includes a colonoscopy that reveals continuous, circumferential inflammatory changes in the rectum. The disease typically begins in the rectum and progresses up the colon over the course of the disease. In comparison to Crohn disease, which is known for having skip lesions, ulcerative colitis is more likely to have circumferential lesions. Treatment depends on the amount of colon affected by the disease. Patients with proctitis or proctosigmoiditis should use a topical 5-aminosalicylic acid to induce remission. Topical 5-aminosalicylic acid is available as a suppository and an enema, with the enema being able to reach a longer section of the colon. Those who do not respond to this therapy should be given a foam and suppository preparation. Oral 5-aminosalicylic acid should be used for those with proctitis or proctosigmoiditis who are unwilling or unable to tolerate topical formulations. Those who fail 5-aminosalicylic acid treatment alone should be given a combination of 5-aminosalicylic acid and topical steroids. If patients have left-sided colitis, extensive colitis, or pancolitis, oral 5-aminosalicylic acid with topical 5-aminosalicylic acid or steroid suppositories or enemas should be used. Oral glucocorticoids are used as a last resort for patients failing combination topical and oral 5-aminosalicylic acid. Patients with isolated proctitis do not need medication to maintain remission, however, all other degrees of colonic involvement should be maintained on topical and oral 5-aminosalicylic acid and the lowest possible dose and frequency.
A 24-year-old woman presents to the clinic complaining of new blisters on her right labia. On exam, there is a cluster of 2-3 mm vesicles with surrounding erythema and edema of the right labia. The area is exquisitely tender. Which test would you order to confirm your clinical diagnosis? APap smear BPotassium hydroxide test CVaricella titer DViral culture
D Viral culture Explanation: Genital herpes is a sexually transmitted infection caused by the herpes simplex virus. There are two types of herpes simplex virus: type 1 and type 2. While it is possible for either type 1 or type 2 to affect both anogenital or oropharyngeal regions, traditionally herpes simplex virus type 1 affects the oropharyngeal region and herpes simplex virus type 2 affects the anogenital region. Herpes simplex virus is transmitted by direct contact with an active lesion or during the lesion shedding phase. Genital herpes typically presents with painful genital vesicles or ulcers with associated site erythema and edema. On average, the incubation period after exposure is four days. While the disease can be diagnosed clinically, a viral culture is typically performed to confirm clinical suspicion. A viral culture for herpes simplex virus is performed by collecting a sample of cells from the active lesion using a sterile swab. A viral culture may only be performed on an active herpetic lesion. Similarly, a Tzanck smear may also only be performed on an active herpetic lesion, however, this test has a low sensitivity and will not differentiate between different herpetic diseases. For inactive lesions, polymerase chain reaction may be required for diagnosis. The sensitivity of this test is greater than viral culture, but polymerase chain reaction is currently not as readily available as a viral culture. Presently, there is no cure for herpes simplex virus and patients can expect recurrences. Recurrences are most common in periods of stress and in immunocompromised patients. Genital herpes is treated with antiviral medication (e.g. valacyclovir, acyclovir) and can be administered in both suppressive and abortive doses. Prevention of sexual transmission is best achieved by abstaining from sexual practices during and after outbreaks as well as during the viral shedding phase.
A 49-year-old man presents to the office complaining of left-sided facial drooping developing over the past day. He denies weakness of extremities, slurred speech, or numbness. Examination reveals inability to close the left eye, disappearance of the left nasolabial fold, and drooping of the corner of the mouth. The patient is not able to wrinkle his forehead on the affected side. Which of the following interventions would be appropriate in this patient? ABegin doxycycline 100 mg twice daily for 28 days BRefer for electromyography CRefer to emergency department for stroke workup DStart prednisone 60 mg daily for seven days plus valacyclovir 1,000 mg three times daily for seven days
D start prednisone 60 mg daily for seven days plus valacyclovir 1,000 mg three times daily for seven days Explanation: Idiopathic facial nerve palsy (Bell palsy) is an acute paralysis of the facial nerve. The facial nerve is responsible for innervation of the facial muscles, taste receptors of the anterior two-thirds of the tongue, and parasympathetic innervation of the lacrimal, submandibular, and sublingual glands. Pregnant women have a greater risk of developing Bell palsy. The most common causes of Bell palsy include herpes simplex virus and herpes zoster. Physical examination demonstrates unilateral facial paralysis. This paralysis manifests as sagging of the affected eyebrow, inability to close the eye, flattening of the nasolabial fold, and drooping of the affected corner of the mouth. Patients may also experience decreased tearing and loss of taste sensation in the affected portion of the tongue. Patients may experience a prodrome of ear pain or difficulty hearing. The diagnosis is clinical. Electromyography can be considered in patients with a complete lesion to determine the degree of axonal loss or nerve degeneration, which may correspond to the potential for recovery. Neuroimaging is not typically ordered in the acute stage unless presentation is atypical. Patients with slow recovery beyond three weeks, no improvement after four months, or facial twitching or spasm preceding the facial weakness should undergo imaging to rule out underlying structural lesion or tumor. Contrast-enhanced CT scan or magnetic resonance imaging of the brain, temporal bone, and parotid gland is recommended. Patients who have negative imaging but complete flaccid paralysis at seven months after onset of symptoms will need to undergo parotid gland biopsy. Lyme disease testing is recommended in individuals who live in Lyme-endemic areas upon initial presentation. The test can be repeated after several weeks if the initial test is negative and suspicions remain high, as Bell palsy can occur prior to the patient becoming seropositive. Oral glucocorticoids, particularly prednisone, and antiviral therapy, such as valacyclovir, are started within the first three days of symptom onset. Due to the inability to fully close the eye, patients are advised to use artificial tears and an eye patch to avoid corneal abrasion or injury. In patients with incomplete recovery, multimodal physical therapy is often utilized, which includes biofeedback, electrical stimulation, and massage. Surgical decompression is not a recommended treatment.
A 2-month-old infant presents to the clinic for a routine well-child examination, and a midsystolic heart murmur is auscultated. An echocardiogram shows a defect between the right and left atria. Which of the following causes this heart murmur? ABlood flow from the left to right atrium BBlood flow from the right to left atrium CIncreased blood flow across the aortic valve DIncreased blood flow across the pulmonic valve
DIncreased blood flow across the pulmonic valve Explanation: An atrial septal defect is an opening in the septum between the left and right atrium. It is the second most common congenital heart defect. Atrial septal defects result in a left-to-right shunting of blood because the pressure is higher in the left atria than the right atria. Therefore, it does not result in deoxygenated blood being sent into systemic circulation. The clinical significance of atrial septal defects varies based on its anatomical location and size and the presence of other congenital cardiac abnormalities. Most atrial septal defects are small and asymptomatic. They are most often identified on routine childhood health care visits when an incidental heart murmur is auscultated. If symptoms do occur, then they vary according to the age of the patient. Infants or toddlers may present with exertional dyspnea, failure to thrive, or recurrent respiratory infections. Adolescents and adults may present with exertional dyspnea, exercise intolerance, palpitations, or atrial dysrhythmias. The murmur associated with an atrial septal defect is due to increased blood flow across the pulmonic valve. The increased blood flow across the pulmonic valve causes a midsystolic murmur that is best heard at the left upper sternal border. In addition, a loud S1 and a split and fixed S2 may also be auscultated. Chest radiograph may show cardiomegaly or increased pulmonary vascularity. Echocardiogram is the definitive study. The majority of atrial septal defects resolve spontaneously. However, larger lesions may require surgical patching.
A patient returns to the clinic to discuss laboratory results from a thyroid panel. The panel revealed TSH 0.23 mIU/L, free T3 1.23 ng/dL, T4 36 pmol/L, elevated antithyroperoxidase antibodies, and presence of thyroid-stimulating immunoglobulin. Which of the following physical exam findings is most likely present? AAsymmetric, hard, fixed goiter BFluctuant unilateral neck mass CMultiple palpable thyroid nodules DRaised, violaceous papules on the shins
DRaised, violaceous papules on the shins Hyperthyroidism is an endocrine disorder caused by elevated serum levels of thyroid hormones (T4 and T3). Graves disease, the most common cause of hyperthyroidism, is an autoimmune disorder of the thyroid gland, which causes increased synthesis and release of thyroid hormones. This condition is more common in women with a typical onset between ages 20 to 40 years. Characteristic clinical findings include infiltrative ophthalmopathy (Graves exophthalmos) and infiltrative dermopathy (pretibial myxedema), which are described as proptosis with tearing and periorbital edema and raised, violaceous papules on the shins, respectively. Other signs and symptoms include anxiety, emotional lability, weakness, tremor, increased deep tendon reflexes, palpitations, thinning hair or hair loss, increased defecation, heat intolerance, sweating, and weight loss despite normal or increased appetite. Physical exam often reveals a thyroid that is symmetrically enlarged with a prominent goiter. Laboratory results indicate elevated thyroid hormone levels with decreased serum thyroid-stimulating hormone. Findings specific to Graves disease include detectable serum thyroid-stimulating immunoglobulins (thyroid-stimulating hormone receptor antibodies). Nonspecific findings include elevated antithyroperoxidase or antithyroglobulin antibodies and antinuclear antibodies. Radioisotope uptake and scanning can aid in diagnosing patients without overt hyperthyroidism, with a high uptake of radioiodine that is diffuse and symmetric in patients with Graves disease. Thyroid ultrasound is useful for investigating goiter and palpable thyroid nodules. Graves ophthalmopathy is best visualized with magnetic resonance imaging and CT scans of the orbits. First-line treatment for any patient with symptomatic thyrotoxicosis involves a beta-blocker (propranolol, metoprolol). Antithyroid drugs include propylthiouracil and methimazole. These drugs offer remission in patients with a high likelihood (e.g., women, mild disease, small goiter, low thyroid antibody levels) and afford rapid control of symptoms in those with moderate or severe Graves disease. Radioactive iodine therapy is a method for definitive control of Graves hyperthyroidism and is appropriate in patients who are unable to undergo surgery or take antithyroid drugs, or who fail to become euthyroid with medical therapy. Women should practice contraceptive methods for at least four to six months following therapy while men require three to four months prior to attempting to conceive with a partner. Following radioactive iodine therapy, patients demonstrate hypothyroidism, which is treated with lifelong thyroid supplementation (levothyroxine). Thyroidectomy is warranted in patients with thyroid malignancy, large nodules, coexisting hyperparathyroidism, moderate-to-severe Graves disease, and those requiring prompt and definitive control of hyperthyroidism. Similar to radioactive treatment, patients are treated with levothyroxine or similar thyroid supplements after surgical resection.
A 50-year-old woman is complaining of progressive solid and liquid dysphagia with regurgitation of undigested food. She has also noticed retrosternal burning and gets the hiccups more frequently. Esophageal manometry reveals aperistalsis in the distal two-thirds of the esophagus accompanied by incomplete relaxation of the lower esophageal sphincter and elevated lower esophageal sphincter resting pressures. Which of the following clinical interventions is associated with the longest duration of symptom relief? ABotulinum toxin injection BOral nitrate therapy CProton pump inhibitor therapy DSurgical myotomy
DSurgical myotomy Explanation: Surgical myotomy , which requires the lower esophageal sphincter muscle fibers to be surgically cut, is the clinical intervention that provides the longest duration of symptom relief in patients with achalasia. This is of importance as many clinical interventions for achalasia are associated with high rates of symptoms recurrence. Disruption of the lower esophageal sphincter muscle fibers with myotomy or balloon dilation provides patients with sustained symptomatic relief, however while it is superior to the other methods suggested in this question, one-third to one-half of all patients will require a repeat procedure after ten years. Compared to other procedures, there is a longer recovery period after surgery and a potential for complications like developing gastroesophageal reflux disease, pneumothorax, perforation, bleeding or infection. Botulinum toxin injection and oral nitrate therapy aim to reduce the pressure at the lower esophageal sphincter but are inferior in terms of duration of associated relief when compared to myotomy and balloon dilation. Achalasia results from a degeneration of nerves in the myenteric plexus located at the distal end of the esophagus. Over time, this results in aperistalsis and poor relaxation of the lower esophageal sphincter. This causes the typical clinical presentation of progressive solid and liquid dysphagia, regurgitation of undigested food or saliva, inability to belch, and increased hiccups because food and liquids are unable to pass easily into the stomach. Patients may also report dyspepsia or retrosternal burning associated with meals. Patients are usually between 20 and 60 years old and have a history of progressive symptoms over the preceding five years. Esophageal manometry is required to make the diagnosis and will reveal aperistalsis of the inferior two-thirds of the esophagus, poor lower esophageal sphincter relaxation and increased pressure at the lower esophageal sphincter. A barium swallow is often ordered with esophageal manometry and reveals a classic "bird beak" appearance reflective of a dilated, aperistaltic distal esophagus and a closed lower esophageal sphincter. Differential diagnoses should include pseudoachalasia, gastroesophageal reflux disease, and esophageal motility disorders such as diffuse esophageal spasm.
A 60-year-old man who is a patient of yours has had two episodes of acute gout in his left first metatarsophalangeal joint in the past six months. His father also had a history of gout, which became debilitating. He is generally healthy and has no history of renal disease. He started taking allopurinol after the first episode of gout. He would like to know what other steps he can take to reduce his risk of further attacks. Which of the following lifestyle interventions is likely to be most effective? AAbstinence from alcohol BLow-protein diet CVitamin C supplementation DWeight loss and maintaining a healthy weight
DWeight loss and maintaining a healthy weight Explanation: Gout is an inflammatory arthropathy caused by deposition of monosodium urate crystals in joints and soft tissues. Increased serum uric acid, or hyperuricemia, is necessary for crystal deposition, but many people with hyperuricemia are asymptomatic. There is a strong genetic association. Other causes include obesity, hypertension, a purine-rich diet, alcohol use, kidney disease (decreased renal excretion of uric acid), and medications, including some diuretics. It is more common in men and postmenopausal women. A flare of gout can be triggered by trauma, surgery, alcohol consumption, fatty foods, or medications. An acute episode typically involves one joint and presents with the rapid onset of severe pain, swelling, erythema, and decreased range of motion. The most common joints involved are the first metatarsophalangeal joint and the knee, followed by the ankle, wrist, and finger joints. X-rays are usually normal except in chronic disease, when they may show joint destruction. Inflammatory markers, such as the erythrocyte sedimentation rate, are typically elevated during an acute episode. The serum uric acid level is not helpful to exclude gout, as it may be high, low, or normal during an attack. Aspiration of the joint and analysis of the synovial fluid reveals needle-shaped, negatively birefringent uric acid crystals and a white blood cell count in the inflammatory range of 20,000 to 80,000 cells per cubic millimeter. Treatment of acute attacks includes the use of nonsteroidal anti-inflammatory medications, oral or intra-articular glucocorticoids, and oral colchicine until the symptoms resolve. Patients should be treated with a uric acid-lowering medication, such as allopurinol and probenecid, after the acute episode as most patients will have a recurrent attack within two years if not treated. In the past, it was recommended that uric acid-lowering medications not be initiated during an acute flare, because of concerns over precipitating another flare or prolonging inflammatory arthritis. However, recent studies have found no difference in pain levels, frequency of gout flares, or levels of acute-phase reactants between patients who started allopurinol versus a placebo during an acute episode of gout. It may take many months to decrease the total accumulation of urate crystals in the body, so patients may be at risk for recurrent attacks even after starting uric acid-lowering medication. In addition to medications, lifestyle modifications such as weight loss and staying at a healthy weight are important in maintaining reduced uric acid levels. The Dietary Approaches to Stop Hypertension (DASH) diet is effective for weight loss and is low in purines. Characteristics of this diet include increased protein from low-fat dairy products, reduced red meat and shellfish, complex carbohydrates instead of simple sugars, and decreased saturated fat.
True or false: individuals with duodenal ulcers need to follow a special diet.
False
True or false: troponin T is more specific for myocardial infarction than troponin I.
False. Troponin I is more specific for myocardial infarction.
Where does atopic dermatitis most commonly present?
Flexural locations.
What auscultatory finding is pathognomonic for pneumomediastinum?
Hamman sign, which is a crunching, crackling sound heard best over the precordium in synchrony with the heartbeat.
What is the most common type of breast cancer?
Infiltrating ductal carcinoma.
What are some examples of end-organ damage seen in patients with hypertensive emergency?
Intracerebral hemorrhage, hypertensive encephalopathy, acute myocardial infarction, aortic dissection, acute left ventricular failure with pulmonary edema, eclampsia, and unstable angina pectoris.
What alternative treatment can be considered in patients who cannot undergo phlebotomy?
Iron chelation.
What is community-acquired respiratory distress syndrome toxin?
It is a virulence factor possessed by Mycoplasma pneumoniae that may play a role in respiratory epithelial cell destruction.
What is the function of the meibomian gland?
It is an exocrine gland that is responsible for the production of meibum, an oily substance that keeps the eyes moistened.
What is the normal diameter of an adult abdominal aorta?
Less than 3.0 cm in diameter.
Which anticoagulant is recommended in pregnant patients with pulmonary embolism or deep vein thrombosis?
Low-molecular-weight heparin (such as enoxaparin).
What is the recommended medication dosing for women with trichomoniasis and human immunodeficiency virus infection?
Metronidazole 500 mg twice per day for five to seven days.
Which over-thecounter medication used at the time of conception could increase the risk of spontaneous abortion?
Nonsteroidal anti-inflammatory drugs (NSAIDs).
When should iodine be administered to patients with a thyroid storm?
One hour after administration of antithyroid medications (polythiouracil, methimazole).
How much time should lapse before obtaining a prostate-specific antigen level when screening for prostate cancer in a patient with recent acute bacterial prostatitis?
One month after resolution of prostatitis.
What skeletal abnormalities are patients with Cushing syndrome predisposed to developing?
Osteonecrosis of the femoral head, osteoporosis, vertebral compression fractures, and lower extremity insufficiency fractures.
What HIV-related illnesses should be considered when the CD4 count is < 200 cells/mcL?
Pneumocystis pneumonia and progressive multifocal leukoencephalopathy.
Which two locations in the esophagus are most likely to be injured by medication-induced esophagitis?
Proximal esophagus due to compression from the aortic arch and the distal esophagus in patients with left atrial enlargement.
Which organisms cause chronic otitis media?
Pseudomonas aeruginosa, Proteus species, and Staphylococcus aureus.
What are the possible electrocardiogram findings of an atrial septal defect?
QRS pattern suggestive of incomplete right bundle branch block.
What is the condition wherein aggressive large-cell lymphoma develops from underlying chronic lymphocytic leukemia?
Richter transformation (Richter syndrome).
Which herbal supplement is often used to treat benign prostatic hypertrophy, and what is its efficacy?
Saw palmetto but double-blind studies have failed to show any benefit in symptoms or prostate volume when compared to placebo.
Which hematologic disease is known to cause acute chest syndrome?
Sickle cell disease.
How long must the hepatitis B surface antigen persist in order for hepatitis B infection to be considered chronic?
Six months.
What are the best ways to prevent pneumonia?
Smoking cessation, getting the annual influenza vaccine, and getting the pneumococcal vaccines when appropriate.
What is word clanging?
Stringing together words which rhyme, alliterate, or have phonetic similarity in a meaningless manner.
What scoring system is used to evaluate for the need of antithrombotic therapy in patients with atrial fibrillation?
The CHA2DS2-VASc score.
Within which ganglion is latent varicella-zoster virus reactivated to result in Ramsey Hunt syndrome?
The geniculate ganglion.
What are some other masses found on the wrist and hand?
The most common solid mass is a giant cell tumor of the tenosynovium. Others include epidermal inclusion cysts, lipomas, rheumatoid nodules, gouty tophi, and synovial sarcomas.
What is a "tail sign" on MRI?
The peripheral tapering of dural thickening of the meningioma.
What is the pathologic mechanism that causes Graves ophthalmopathy?
Thyroid-stimulating antibodies increase secretion of hydrophilic glycosaminoglycans (e.g., hyaluronic acid) that infiltrate the extraocular muscles.
Which two pathogens are responsible for the majority of onychomycosis cases?
Trichophyton rubrum and Trichophyton mentagrophytes.
What is the most common type of von Willebrand disease?
Type 1 von Willebrand disease is the most common type, accounting for approximately 75% of all cases of von Willebrand disease.
What are the two most common ocular extraintestinal manifestations of ulcerative colitis?
Uveitis and episcleritis.
What is the appropriate treatment for a new herpes simplex vaginalis infection in a nulliparous woman?
Valacyclovir
What skin condition presents as dew drops on a rose petal?
Varicella zoster
What is the most common cause of sinusitis overall?
Viruses